% LaTeX source for Answers to Bayesian Statistics: An Introduction (4th edn)

\documentclass[oneside]{book}
\usepackage{amsmath}
\usepackage{amssymb}
\usepackage{amsbsy}
\usepackage{makeidx}
\usepackage{epsf}
\usepackage{euscript}

\setcounter{secnumdepth}{1}

\setcounter{tocdepth}{1}

% Set up environment for exercises at ends of chapters
\newcounter{qno}
\newcommand{\startqs}{\setcounter{qno}{0}\vspace{-1.5\baselineskip}}
\newcommand{\nextq}
    {\vspace{1.5\baselineskip}\noindent\addtocounter{qno}{1}\arabic{qno}.\quad}

% Allow for blank lines
\newcommand{\blankline}{\vspace{\baselineskip}\noindent}

% Define digitwidth and dotwidth (TeXbook p. 241)
\newdimen\digitwidth
\setbox0=\hbox{\rm0}
\digitwidth=\wd0
\newdimen\dotwidth
\setbox0=\hbox{\rm.}
\dotwidth=\wd0

% Notation for vectors, matrices, estimates, random variables and sample means
\newcommand{\vect}{\boldsymbol}
\newcommand{\matr}{\boldsymbol}
\newcommand{\est}{\widehat}
\newcommand{\random}{\widetilde}
\newcommand{\mean}{\overline}

% Notation for dots in subscripts
\newcommand {\bdot}{\hbox{\Huge .}}
\newcommand {\dotdot}{{\hbox{\Huge .}\kern-0.1667em\hbox{\Huge .}}}
\newcommand {\onedot}{1\kern-0.1667em\bdot}
\newcommand {\twodot}{2\kern-0.1667em\bdot}
\newcommand {\idot}{i\kern-0.1667em\bdot}
\newcommand {\jdot}{j\kern-0.1667em\bdot}
\newcommand {\mdot}{m\kern-0.1667em\bdot}
\newcommand {\dotj}{\kern-0.1667em\bdot\kern-0.1667em j}

% Define sech, arc sin and arc cos
\newcommand{\sech}{\operatorname{sech}}
\renewcommand{\arcsin}{\operatorname{arc\,sin}}
\renewcommand{\arccos}{\operatorname{arc\,cos}}

% Define Probability, Expectation, Variance, Covariance, Median, Mode
\renewcommand{\Pr}{\mbox{$\mathsf P$}}
\newcommand{\E}{\mbox{$\mathsf E$}}
\newcommand{\Var}{\mbox{$\mathcal V$}}
\newcommand{\Cov}{\mbox{$\mathcal C$}}
\newcommand{\median}{\mbox{median\,}}
\newcommand{\mode}{\mbox{mode\,}}

\newcommand{\MCMC}{\mbox{\text{MCMC}}}

% Define notation for evidence
\newcommand{\Ev}{\mbox{Ev}}

% Notation for the R project
\newcommand{\R}{\textsf{R}}

% Script I for a (Kullback-Leibler) information measure
\newcommand{\I}{\mbox{$\EuScript I$}}

% Define small common fractions for use in display formulae
\newcommand{\half}{\mbox{$\frac{1}{2}$}}
\newcommand{\smallhalf}{\mbox{\small$\frac{1}{2}$}}
\newcommand{\quarter}{\mbox{$\frac{1}{4}$}}
\newcommand{\threequarters}{\mbox{$\frac{3}{4}$}}
\newcommand{\third}{\mbox{$\frac{1}{3}$}}
\newcommand{\twothirds}{\mbox{$\frac{2}{3}$}}
\newcommand{\ninth}{\mbox{$\frac{1}{9}$}}
\newcommand{\twofifths}{\mbox{$\frac{2}{5}$}}

% Alternative notation for fractions (TeXbook, exercise 11.6)
\newcommand{\slopefrac}[2]{\leavevmode\kern.1em
\raise .5ex\hbox{\the\scriptfont0 #1}\kern-.1em
/\kern-.15em\lower .25ex\hbox{\the\scriptfont0 #2}}

% Notation for beta function
\newcommand{\Betafn}{\mbox{B}}

% Define names of distributions
\newcommand{\N}{\mbox{N}}              % A.1
\newcommand{\G}{\mbox{G}}              % A.4
\newcommand{\Ex}{\mbox{E}}             % A.4
\renewcommand{\t}{\mbox{t}}            % A.8
\newcommand{\Be}{\mbox{Be}}            % A.10
\newcommand{\B}{\mbox{B}}              % A.11
\renewcommand{\P}{\mbox{P}}            % A.12
\newcommand{\NB}{\mbox{NB}}            % A.13
\renewcommand{\H}{\mbox{H}}            % A.14
\newcommand{\U}{\mbox{U}}              % A.15
\newcommand{\UD}{\mbox{UD}}            % A.15
\newcommand{\Pa}{\mbox{Pa}}            % A.16
\newcommand{\Pabb}{\mbox{Pabb}}        % A.16
\newcommand{\M}{\mbox{M}}              % A.17
\newcommand{\BF}{\mbox{BF}}            % A.18
\newcommand{\F}{\mbox{F}}              % A.19
\newcommand{\z}{\mbox{z}}              % A.20
\newcommand{\C}{\mbox{C}}              % A.21

% Define some common bold symbols
\newcommand{\balpha}{\mbox{$\boldsymbol\alpha$}}
\newcommand{\bbeta}{\mbox{$\boldsymbol\beta$}}
\newcommand{\beeta}{\mbox{$\boldsymbol\eta$}}
\newcommand{\btheta}{\mbox{$\boldsymbol\theta$}}
\newcommand{\bkappa}{\mbox{$\boldsymbol\kappa$}}
\newcommand{\blambda}{\mbox{$\boldsymbol\lambda$}}
\newcommand{\bmu}{\mbox{$\boldsymbol\mu$}}
\newcommand{\bnu}{\mbox{$\boldsymbol\nu$}}
\newcommand{\bpi}{\mbox{$\boldsymbol\pi$}}
\newcommand{\btau}{\mbox{$\boldsymbol\tau$}}
\newcommand{\bzero}{\mbox{$\boldsymbol0$}}
\newcommand{\BOmega}{\mbox{$\boldsymbol\Omega$}}

% Further bold symbols for use in connection with hierarchical models
\newcommand {\bpiem}{\mbox{\boldmath $\pi^{EM}$}}
\newcommand {\bhtheta}{\mbox{\boldmath $\est\theta$}}
\newcommand {\bhthetao}{\mbox{\boldmath $\est\theta^{\mbox{\scriptsize\it0}}$}}
\newcommand {\bhthetajs}{\mbox{\boldmath $\est\theta^{JS}$}}
\newcommand {\bhthetajsplus}{\mbox{\boldmath $\est\theta^{JS^{{}_+}}$}}
\newcommand {\bhthetaem}{\mbox{\boldmath $\est\theta^{EM}$}}
\newcommand {\bhthetab}{\mbox{\boldmath $\est\theta^{B}$}}
\newcommand {\bhthetaeb}{\mbox{\boldmath $\est\theta^{EB}$}}
\newcommand {\thetabar}{\mbox{$\mean\theta$}}
\newcommand {\bphi}{\mbox{\boldmath $\phi$}}
\newcommand {\BPhi}{\mbox{\boldmath $\Phi$}}
\newcommand {\bpsi}{\mbox{\boldmath $\psi$}}
\newcommand {\BPsi}{\mbox{\boldmath $\Psi$}}
\newcommand {\BSigma}{\mbox{\boldmath $\Sigma$}}

% Define transpose for matrix theory
\newcommand{\transpose}{\mbox{${}^{\text{T}}$}}

% Define differentials with roman d and thin space before
\renewcommand{\d}{\mbox{d}}
\newcommand{\dF}{\,\mbox{\d$F$}}
\newcommand{\dt}{\,\mbox{\d$t$}}
\newcommand{\du}{\,\mbox{\d$u$}}
\newcommand{\dU}{\,\mbox{\d$U$}}
\newcommand{\dx}{\,\mbox{\d$x$}}
\newcommand{\dy}{\,\mbox{\d$y$}}
\newcommand{\dz}{\,\mbox{\d$z$}}
\newcommand{\dgamma}{\,\mbox{\d$\gamma$}}
\newcommand{\dzeta}{\,\mbox{\d$\zeta$}}
\newcommand{\deta}{\,\mbox{d$\eta$}}
\newcommand{\dtheta}{\,\mbox{\d$\theta$}}
\newcommand{\dbtheta}{\,\mbox{\d$\boldsymbol\theta$}}
\newcommand{\dkappa}{\,\mbox{\d$\kappa$}}
\newcommand{\dlambda}{\,\mbox{\d$\lambda$}}
\newcommand{\dLambda}{\,\mbox{\d$\Lambda$}}
\newcommand{\dmu}{\,\mbox{\d$\mu$}}
\newcommand{\dbmu}{\,\mbox{\d$\bmu$}}
\newcommand{\drho}{\,\mbox{\d$\rho$}}
\newcommand{\dpi}{\,\mbox{\d$\pi$}}
\newcommand{\dxi}{\,\mbox{\d$\xi$}}
\newcommand{\dphi}{\,\mbox{\d$\phi$}}
\newcommand{\dpsi}{\,\mbox{\d$\psi$}}
\newcommand{\domega}{\,\mbox{\d$\omega$}}

% Hyp for hypothesis
\newcommand{\Hyp}{\mbox{H}}

% Blackboard bold Z for the integers
\newcommand{\Z}{\mbox{$\mathbb Z$}}

% Script X for a set of possible observations
\newcommand{\X}{\mbox{$\mathcal X$}}

% EM, GEM, E-step and M-step for the EM algorithm
\newcommand{\EM}{\mbox{\textit{EM}\ }}
\newcommand{\GEM}{\mbox{\textit{GEM}\ }}
\newcommand{\Estep}{\mbox{\textit{E}-step\ }}
\newcommand{\Mstep}{\mbox{\textit{M}-step\ }}

% Omit the word Chapter at the start of chapters
\renewcommand{\chaptername}{}

\begin{document}

\pagestyle{plain}

\appendix

\setcounter{chapter}{8}

\section{Exercises on Chapter \arabic{section}}

\setcounter{qno}{0}

\nextq A card came is played with 52 cards divided equally between four 
     players, North, South, East and West, all arrangements being
     equally likely.  Thirteen of the cards are referred to as trumps.  
     If you know that North and South have ten trumps between them, 
     what is the probability that all three remaining trumps are in the 
     same hand?  If it is known that the king of trumps is included 
     among the other three, what is the probability that one player has 
     the king and the other the remaining two trumps?

\nextq\!\!\!\!(a) Under what circumstances is an event $A$ independent
     of itself?

\begin{description}
  \item[\quad(b)] By considering events concerned with independent
     tosses of a red die and a blue die, or otherwise. give examples 
     of events $A$, $B$ and $C$ which are not independent, but 
     nevertheless are such that every pair of them is independent.

  \item[\quad(c)]By considering events concerned with three independent
     tosses of a coin and supposing that $A$ and $B$ both represent 
     tossing a head on the first trial, give examples of events $A$, 
     $B$ and $C$ which are such that $\Pr(ABC)=\Pr(A)\Pr(B)\Pr(C)$ 
     although no pair of them is independent.
\end{description}

\nextq Whether certain mice 
     are black or brown depends on a pair of genes, 
     each of which is either $B$ or $b$.  If both members of the pair
     are alike, the mouse is said to be homozygous, and if they are
     different it is said to be heterozygous.  The mouse is brown 
     only if it is homozygous 
     $bb$.  The offspring of a pair of mice have two such genes, one
     from each parent, and if the parent is heterozygous, the 
     inherited gene is equally 
     likely to be $B$ or $b$.  Suppose that a black mouse results from a
     mating between two heterozygotes.

\begin{description}
\item[\quad(a)] What are the probabilities that this mouse is homozygous
     and that it is heterozygous?
\end{description}

     Now suppose that this mouse is mated with a brown mouse, resulting
     in seven offspring, all of which turn out to be black.

\begin{description}
\item[\quad(b)] Use Bayes' Theorem to find the probability that the
     black mouse was homozygous $BB$.

\item[\quad(c)] Recalculate the same probability by regarding the seven 
     offspring as seven observations made sequentially, treating the
     posterior after each observation as the prior for the next (cf.\ 
     Fisher, 1959, Section II.2).
\end{description}

\nextq The example on Bayes' Theorem in Section 1.2
     concerning the biology of twins 
     was based on the assumption that births of boys and girls occur 
     equally frequently, and yet it has been known for a very long time
     that fewer girls are born than boys (cf.\ Arbuthnot, 1710).  
     Suppose that the probability of a girl is $p$, so that
\[
\begin{array}{lll}
     \Pr(GG|M)=p,   &\Pr(BB|M)=1 - p,     &\Pr(GB|M)=0,               
\\
     \Pr(GG|D)=p^2, &\Pr(BB|D)=(1 - p)^2, &\Pr(GB|D)=2p(1 - p).
\end{array}
\]
     Find the proportion of monozygotic twins in the whole population
     of twins in terms of $p$ and the sex distribution among all twins.


\nextq Suppose a red and a blue die are tossed.  Let $x$ be the sum of
     the number showing on the red die and twice the number showing on
     the blue 
     die.  Find the density function and the distribution function of
     $x$.

\nextq Suppose that $k\sim\B(n,\pi)$ where $n$ is large and $\pi$ is
     small but $n\pi=\lambda$ has an intermediate value.  Use the
     exponential
     limit $(1+x/n)^n\to\text{e}^x$ to show that 
     $\Pr(k=0)\cong \text{e}^{-\lambda}$ and 
     $\Pr(k=1)\cong \lambda\text{e}^{-\lambda}$.  Extend this result to 
     show that $k$ is such that
\[      p(k) \cong \frac{\lambda^k}{k!}\exp(-\lambda) \]
     that is, $k$ is approximately distributed as a Poisson variable of
     mean $\lambda$ (cf.\ Appendix A).

\nextq Suppose that $m$ and $n$ have independent Poisson distributions
     of means $\lambda$ and $\mu$ respectively (see question 
     6 and that $k=m+n$.
     has a chi-squared density 
     on one degree of freedom as defined in Appendix A.

\nextq Modify the formula for the density of a one-to-one funtion $g(x)$
     of a random variable $x$ to find an expression for the density of
     $x^2$ in terms of that of $x$, in both the continuous and discrete
     case.  Hence show that the square of a standard normal distribution
     has a chi-squared distribution on one degree of freedom as defined
     in Appendix A.

\nextq Suppose that $x_1, x_2, \dots, x_n$ are independently and all
     have the 
     same continuous distribution, with density $f(x)$ and distribution 
     function $F(x)$.  Find the distribution functions of
\[   M  = \max \{x_1, x_2, \dots, x_n\} \quad\text{and}\quad 
     m  = \min \{x_1, x_2, \dots, x_n\} \]
     in terms of $F(x)$, and so find expressions for the density
     functions of $M$ and $m$.

\nextq Suppose that $u$ and $v$ are independently uniformly distributed 
     on the interval [0, 1], so that the divide the interval into three
     sub-intervals.  Find the joint density function of the lengths of
     the first two sub-intervals.

\nextq Show that two continuous random variables $x$ and $y$ are
     independent (that is, $p(x, y)=p(x)p(y)$ for all $x$ and $y$) if
     and only if their joint distribution function $F(x, y)$ satisfies
     $F(x, y)=F(x)F(y)$ for all $x$ and $y$.  Prove that the same thing
     is true for discrete random variables.  [This is an example of a
     result which is easier to prove in the continuous case.]

\nextq Suppose that the random variable $x$ has a negative binomial 
     distribution 
     $\NB(n, \pi)$ of index $n$ and parameter $\pi$, so that
\[               p(x)  =   \binom{n+x-1}{x}  \pi^n (1 - \pi)^x \]
     Find the mean and variance of $x$ and check that your answer agrees
     with that given in Appendix A.

\nextq A random variable $X$ is said to have a chi-squared distribution
     on $\nu$ degrees of freedom if it has the same distribution as
\[      Z_1^2+Z_2^2+\dots+Z_{\nu}^2 \]
     where $Z_1$, $Z_2$, $\dots$, $Z_{\nu}$ are independent standard
     normal variates.  Use the facts that $\E Z_i=0$, $\E Z_i^2=1$ and 
     $\E Z_i^4=3$ to find the mean and variance of $X$.  Confirm these
     values using the probability density of $X$, which is
\[     p(X)=\frac{1}{2^{\nu/2}\Gamma(\nu/2)}X^{\nu/2-1}\exp(-\half X)
       \qquad(0 < X < \infty) \]
     (see Appendix A).

\nextq The \textit{skewness} of a random variable $x$ is defined as  
     $\gamma_1 = \mu_3/(\mu_2)^{\frac{3}{2}}$ where
\[               \mu_n  =  \E (x  -  \E x)^n \]
     (but note that some authors work in terms of  $\beta_1 = 
     \gamma_1^2$).  
     Find the skewness of a random variable $X$ with a binomial
     distribution $B(n, \pi)$ of index $n$ and parameter $\pi$.
     
\nextq Suppose that a continuous random variable $X$ has mean $\mu$ and
     variance $\phi$.  By writing
\[     \phi = \int (x-\mu)^2 p(x)\dx \geqslant
              \int_{\{x;\,|x-\mu|\geqslant c\}} (x-\mu)^2 p(x)\dx \]
     and using a lower bound for the integrand in the latter integral,
     prove that
\[     \Pr(|x-\mu|\geqslant c)\leqslant\frac{\phi}{c^2}. \]
     Show that the result also holds for discrete random variables.
     [This result is known as \v Ceby\v sev's Inequality (the name is
     spelt in many other ways, including Chebyshev and Tchebycheff).]

\nextq Suppose that $x$ and $y$ are such that
\[    \Pr(x=0, y=1)=\Pr(x=0, y=-1)=\Pr(x=1, y=0)=\Pr(x=-1,
      y=0)=\quarter. \]
     Show that $x$ and $y$ are uncorrelated but that they are
     \textit{not} independent.

\nextq Let $x$ and $y$ have a bivariate normal distribution 
     and suppose that 
     $x$ and $y$ both have mean 0 and variance 1, so that their marginal
     distributions are standard normal and their joint density is
\[   p(x, y)  =  \left\{2\pi\sqrt{(1 - \rho^2)}\right\}^{-1}
      \exp \left\{- \half(x^2 - 2\rho xy + y^2)/(1 - \rho^2) \right\}.
\]
     Show that if the correlation coefficient between $x$ and $y$ is 
     $\rho$, then that between $x^2$ and $y^2$ is $\rho^2$.

\nextq Suppose that $x$ has a Poisson distribution 
     (see question 6) $\P(\lambda)$ of mean 
     $\lambda$ and that, for given $x$, $y$ has a binomial distribution 
     $\B(x, \pi)$ of index $x$ and parameter $\pi$.

\begin{description}
\item[\quad(a)] Show that the unconditional distribution of $y$ is
     Poisson of mean
\[   \lambda\pi = \E_{\random x} 
     \E_{\random y|\random x}(\random y|\random x).                  \]

\item[\quad(b)] Verify that the formula
\[   \Var\,\random y = 
   \E_{\random x}\Var_{\random y|\random x}(\random y|\random x)
   +\Var_{\random x}\E_{\random y|\random x}(\random y|\random x)
\]
derived in Section 1.5 holds in this case.
\end{description}

\nextq Define
\[ I=\int_{0}^{\infty}\exp(-\half z^2)\,dz \]
     and show (by setting $z=xy$ and then substituting $z$ for $y$) that 
\[ I=\int_{0}^{\infty}\exp(-\half(xy)^2)\,y\,dx
    =\int_{0}^{\infty}\exp(-\half(zx)^2)\,z\,dx. \]
     Deduce that
\[ I^2=\int_{0}^{\infty}\int_{0}^{\infty}
        \exp\{-\half(x^2+1)z^2\}\,z\,dz\,dx.             \]
     By substituting $(1+x^2)z^2=2t$ so that $z\,dz=dt/(1+x^2)$ show 
     that $I=\sqrt{\pi/2}$ so that the density of the standard normal 
     distribution as defined in Section 1.3 does integrate to unity 
     and so is indeed a density.  (This method is due to Laplace, 1812, 
     Section 24.)

\section{Exercises on Chapter \arabic{section}}

\setcounter{qno}{0}

\nextq Suppose that $k\sim\B(n,\pi)$.  Find the standardized likelihood
     as a function of $\pi$ for given $k$.  Which of the distributions
     listed in Appendix A does this represent?


\nextq Suppose we are given the twelve observations from a normal
     distribution:

\begin{center}
  15.644,\ \ \ 16.437,\ \ \ 17.287,\ \ \ 14.448,\ \ \ 15.308,\ \ \
  15.169, \\
  18.123,\ \ \ 17.635,\ \ \ 17.259,\ \ \ 16.311,\ \ \ 15.390,\ \ \
  17.252. \\
\end{center}
     and we are told that the variance $\phi = 1$.  Find a 90\% HDR for
     the posterior distribution of the mean assuming the usual reference
     prior.


\nextq With the same data as in the previous question, what is the
     predictive distribution for a possible future observation $x$?


\nextq A random sample of size $n$ is to be taken from an
     $\N(\theta,\phi)$ 
     distribution where $\phi$ is known.  How large must $n$ be to
     reduce the posterior variance of $\phi$ to the fraction $\phi/k$ 
     of its original value (where $k > 1$)?


\nextq Your prior beliefs about a quantity $\theta$ are such that
\[       p(\theta) = \left\{\begin{array}{ll} 1 & (\theta \geqslant 0)  
 \\
                                              0 & (\theta <  0).   
                          \end{array}\right.                            
 \]
     A random sample of size 25 is taken from an $\N(\theta, 1)$
     distribution 
     and the mean of the observations is observed to be 0.33.  Find a
     95\% HDR for $\theta$.


\nextq Suppose that you have prior beliefs about an unknown quantity
     $\theta$ which can be approximated by an $\N(\lambda, \phi)$ 
     distribution, while my beliefs can be approximated by an
     $\N(\mu,\psi)$ distribution.  Suppose further that the reasons 
     that have led us to these conclusions do not overlap with one 
     another.  What distribution should represent our beliefs about 
     $\theta$ when we take into account 
     all the information available to both of us?


\nextq Prove the theorem quoted without proof in Section 
     2.4.

\nextq Under what circumstances can a likelihood arising from a
     distribution in the exponential family be expressed in data
     translated form?

\nextq Suppose that you are interested in investigating how variable the
     performance of schoolchildren on a new mathematics test, and that
     you begin by trying this test out on children in twelve similar
     schools.  It turns out that the average standard deviation is 
     about 10 marks.  You then want to try the test on a thirteenth 
     school, which is fairly 
     similar to those you have already investigated, and you reckon that
     the data on the other schools gives you a prior for the variance in
     this new school which has a mean of 100 and is worth 8 direct 
     observations on the school.  What is the posterior distribution for
     the variance if you then observe a sample of size 30 from the
     school of which the standard deviation is 13.2?  Give an interval 
     in which the variance lies with 90\% posterior probability.


\nextq The following are the dried weights of a number of plants (in
     grammes) from a batch of seeds:
\begin{center}
     4.17,\ \ 5.58,\ \ 5.18,\ \ 6.11,\ \ 4.50,\ \ 4.61,\ \ 5.17,\ \
     4.53,\ \ 5.33,\ \ 5.14.
\end{center}
     Give 90\% HDRs for the mean and variance of the population from
     which they come.


\nextq Find a sufficient statistic 
     for $\mu$ given an $n$-sample 
     $\vect x = (x_1, x_2, \dots, x_n)$ from the exponential
distribution
\[         p(x|\mu)  =  \mu^{-1}\exp (- x/\mu)\qquad(0  <  x  < \infty )
\]
     where the parameter $\mu$ can take any value in $0 < \mu < \infty$.


\nextq Find a (two-dimensional) sufficient statistic 
     for $(\alpha, \beta)$ 
     given an $n$-sample $\vect x = (x_1, x_2, \dots, x_n)$ from the 
     two-parameter gamma distribution
\[     p(x|\alpha,\beta)=\{\beta^\alpha\Gamma(\alpha)\}^{-1}
            x^{\alpha-1}\exp (- x/\beta)\qquad(0  <  x <  \infty)    \]
     where the parameters $\alpha$ and $\beta$ can take any values in 
     $0 < \alpha < \infty$,  $0 < \beta < \infty$.


\nextq Find a family of conjugate priors for the likelihood 
     $l(\beta|x) = p(x\,|\,\alpha,\beta)$ where $p(x\,|\,\alpha,\beta)$ 
     is as in the previous question, but $\alpha$ is known.


\nextq Show that the tangent of a random angle (that is, one which is
     uniformly distributed on $[0, 2\pi)$) has a Cauchy distribution
     $\C(0,1)$.


\nextq Suppose that the vector $\vect x = (x, y, z)$ has a trinomial 
     distribution 
     depending on the index $n$ and the parameter 
     $\bpi = (\pi,\rho,\sigma)$ where $\pi+\rho+\sigma=1$, that is
\[      p(x|\bpi) = 
                \frac{n!}{x!\,y!\,z!}\pi^x\rho^y\sigma^z\qquad(x+y+z=n).
  \]
     Show that this distribution is in the two-parameter exponential
     family.

\nextq Suppose that the results of a certain test are known, on the
     basis 
     of general theory, to be normally distributed about the same mean
     $\mu$ with the same variance $\phi$, neither of which is known.  
     Suppose further that your prior beliefs about $(\mu, \phi)$ can be 
     represented by a normal/chi-squared distribution with
\[     \nu_0  =  4,\qquad S_0  = 350,\qquad  n_0 = 1,\qquad \theta_0  =
       85. \]
     Now suppose that 100 observations are obtained from the population 
     with mean 89 and sample variance $s^2 = 30$.  Find the posterior 
     distribution of $(\mu, \phi)$.  Compare 50\% prior and posterior 
     HDRs for $\mu$.

\nextq Suppose that your prior for $\theta$ is a $\twothirds:\third$
     mixture of $\N(0,1)$ and $\N(1,1)$ and that a single observation
     $x\sim\N(\theta, 1)$ turns out to equal 2.  What is your posterior
     probability that $\theta>1$?


\nextq Establish the formula
\[  (n_0^{-1}+n^{-1})^{-1}(\mean x-\theta_0)^2=
     n\mean x^2+n_0\theta_0^2-n_1\theta_1^2                             
\]
     where $n_1=n_0+n$ and $\theta_1=(n_0\theta_0+n\mean x)/n_1$, which
     was quoted in Section 2.13 as providing a formula for the parameter
     $S_1$ of the posterior distribution in the case where both mean and
     variance are unknown which is less susceptible to rounding errors.

\section{Exercises on Chapter \arabic{section}}

\setcounter{qno}{0}

\nextq Laplace 
     claimed that the probability that an event which has occurred 
     $n$ times, and has not hitherto failed, will occur again is 
     $(n + 1)/(n + 2)$ [see Laplace (1774)], which is sometimes known 
     as \textit{Laplace's rule of succession}.  Suggest grounds for 
     this assertion.

\nextq Find a suitable interval of 90\% posterior probability to quote
     in a case when your posterior distribution for an unknown parameter 
     $\pi$ is $\Be(20, 12)$, 
     and compare this interval with similar 
     intervals for the cases of $\Be(20.5, 12.5)$ and $\Be(21, 13)$ 
     posteriors.  Comment on the relevance of the results to the choice 
     of a reference prior for the binomial distribution.

\nextq Suppose that your prior beliefs about the probability $\pi$ of
     success in Bernoulli trials have mean $1/3$ and variance $1/32$.
     Give a 95\% posterior HDR for $\pi$ given that you have observed
     8 successes in 20 trials.

\nextq Suppose that you have a prior distribution for the probability
     $\pi$ of success in a certain kind of gambling game which has mean 
     0.4, and that you regard your prior information as equivalent to 12 
     trials.  You then play the game 25 times and win 12 times.  What 
     is your posterior distribution for $\pi$?

\nextq Suppose that you are interested in the proportion of females in a
     certain organisation and that as a first step in your investigation
     you intend to find out the sex of the first 11 members on the
     membership list.  Before doing so, you have prior beliefs which you
     regard as equivalent to 25\% of this data, and your prior beliefs
     suggest that a third of the membership is female.
     
     Suggest a suitable prior distribution and find its standard
     deviation.
     
     Suppose that 3 of the first 11 members turn out to be female; find
     your posterior distribution and give a 50\% posterior HDR for this 
     distribution.
     
     Find the mean, median and mode of the posterior distribution.
      
     Would it surprise you to learn that in fact 86 of the total number
     of 433 members are female?

\nextq Show that if $g(x) = \sinh^{-1} \sqrt{(x/n)}$ then
\[               g'(x) = \half n^{-1} [(x/n)\{1 +
                 (x/n)\}]^{-\frac{1}{2}}. \]
     Deduce that if $x \sim \NB(n, \pi)$ has a negative binomial 
     distribution 
     of index n and parameter $\pi$ and $z = g(x)$ then 
     $\E z \cong \sinh^{-1} \sqrt{(x/n)}$ and $\Var z \cong 1/4 n$. 
     What does this suggest as a reference prior for $\pi$?

\nextq The following data were collected by von Bortkiewicz (1898) on
     the number of men killed by a horse 
     in certain Prussian army corps in 
     twenty years, the unit being one army corps for one year:
\begin{center}
\begin{tabular}{lrrrrrl}
   Number of deaths:   &0      &1      &2    &3     &4      &5 and more
\\
   Number of units:    &144    &91     &32   &11    &2      &0.
\end{tabular}
\end{center}
     Give an interval in which the mean number $\lambda$ of such deaths
     in a particular army corps in a particular year lies with 95\%
     probability.

\nextq Recalculate the answer to the previous question assuming that you
     had a prior distribution for $\lambda$ of mean 0.66 and standard
     deviation 0.115.

\nextq Find the Jeffreys prior for the parameter $\alpha$ of the
     Maxwell distribution
\[ p(x|\alpha)=\sqrt{\frac{2}{\pi}}\alpha^{3/2}x^2\exp(-\half\alpha x^2)
\]
     and find a transformation of this parameter in which the
     corresponding prior is uniform.
     
\nextq Use the two-dimensional version of Jeffreys' rule to determine a
     prior for the trinomial distribution
\[     p(x, y, z|\pi,\rho)\propto\pi^x\rho^y(1-\pi-\rho)^z. \]
(cf.\ question 15 on Chapter 2).

\nextq Suppose that $x$ has a Pareto distribution 
     $\Pa(\xi,\gamma)$ where $\xi$ 
     is known but $\gamma$ is unknown, that is,
\[    p(x|\gamma) = \gamma\xi^\gamma x^{-\gamma-1} I_{(\xi,\infty)}(x). 
\]
     Use Jeffreys' rule 
     to find a suitable reference prior for $\gamma$.

\nextq Consider a uniform distribution 
     with $\gamma = 2$.  How large a random sample must 
     be taken from the uniform distribution in order that the
     coefficient of variation (that is, the standard deviation 
     divided by the mean) of 
     the length $\beta - \alpha$ of the interval should be reduced to 
     0.01 or less?

\nextq Suppose that observations $x_1$, $x_2$, $\dots$, $x_n$ are
     available from a density
\[       p(x|\theta)=(c+1)\theta^{-(c+1)}x^c\qquad(0 < x < \theta). \]
     Explain how you would make inferences about the parameter $\theta$ 
     using a conjugate prior.
     
\nextq What could you conclude if you observed \textit{two} tramcars
     numbered, say, 71 and 100?

\nextq In Section 3.8 we discussed Newcomb's
     observation 
     that the front pages of a well-used table of logarithms tend to get

     dirtier than the back pages do.  What if we had an
\textit{antilogarithm} 
     table, that is, a table giving the value of $x$ when $\log_{10} x$
     is given?  Which pages of such a table would be the dirtiest?

\nextq We sometimes investigate distributions on a circle (for 
     example, von Mises' distribution which is discussed in Section 
     3.9 on ``The circular normal distribution'').
     Find a Haar prior for a location parameter on the circle (such 
     as $\mu$ in the case of von Mises' distribution).

\nextq Suppose that the prior distribution $p(\mu, \sigma)$ for the
     parameters $\mu$ and $\sigma$ of a Cauchy distribution
\[     p(x|\mu, \sigma)=\frac{1}{\pi}\frac{\sigma}{\sigma^2+(x-\mu)^2}  
\]
     is uniform in $\mu$ and $\sigma$, and that two observations 
     $x_1 = 2$ and $x_2 = 6$ are available from this distribution.  
     Calculate the 
     value of the posterior density $p(\mu, \sigma|\vect x)$ (ignoring 
     the factor $1/\pi^2$) to two decimal places for $\mu = 0, 2, 4, 6, 8$
     and $\sigma = 1, 2, 3, 4, 5$.  Use Simpson's rule to approximate the 
     posterior marginal density of $\mu$, and hence go on to find an 
     approximation to the posterior probability that $3 < \mu < 5$.

\nextq Show that if the log-likelihood $L(\theta|x)$ is a concave
     function of $\theta$ for each scalar $x$ (that is, 
     $L''(\theta|x) \leqslant 0$
     for all $\theta$), then the likelihood function $L(\theta|\vect x)$
     for $\theta$ given an $n$-sample $\vect x = (x_1, x_2, \dots, x_n)$
     has a unique maximum.  Prove that this is the case if the
     observations $x_i$ come from a logistic density
\[ p(\vect x|\theta)=
            \exp(\theta-x)/\{1+\exp(\theta-x)\}^2\qquad(-\infty < x < \infty) \]
     where $\theta$ is an unknown real parameter.  Fill in the details
     of the Newton-Raphson method and the method of scoring for finding 
     the position of the maximum, and suggest a suitable starting point 
     for the algorithms.

\blankline
     [In many applications of Gibbs sampling, which we consider later 
     in Section 9.4, all full conditional densities are 
     log-concave (see Gilks \textit{et al.}, 1996, Section 5.3.3), so
     the study of such densities is of real interest.]

\nextq Show that if an experiment consists of two observations, then the
     total information it provides is the information provided by
     one observation plus the mean amount provided by the second given
     the first.

\nextq Find the entropy $H\{p(\theta)\}$ of a (negative) exponential
     distribution with density
     $p(\theta)=\beta^{-1}\exp(-\theta/\beta)$.

\section{Exercises on Chapter \arabic{section}}

\setcounter{qno}{0}

\nextq Show that if the prior probability $\pi_0$ of a hypothesis is
     close to unity, then the posterior probability $p_0$ satisfies
     $1-p_0\cong(1-\pi_0)B^{-1}$ and more exactly
     $1-p_0\cong(1-\pi_0)B^{-1}+(1-\pi_0)^2(B^{-1}-B^{-2})$.

\nextq Watkins (1986, Section 13.3) reports that theory predicted the 
     existence of a Z 
     particle of mass $93.3 \pm 0.9$ GeV, while first 
     experimental results showed its mass to be $93.0 \pm 1.8$ GeV. 
     Find the prior and posterior odds and the Bayes ratio for the 
     hypothesis that its mass is less than 93.0 GeV.

\nextq An experimental station wishes to test whether a growth hormone
     will increase the yield of wheat above the average value of 100 
     units per plot produced under currently standard conditions.  
     Twelve plots treated with the hormone give the yields:
\[
     140,\quad 103,\quad 73,\quad 171,\quad 137,\quad 91,\quad 81,\quad 
     157,\quad 146,\quad 69,\quad 121,\quad 134.
\]
     Find the $P$-value for the hypothesis under consideration.

\nextq In a genetic 
     experiment, theory predicts that if two genes are on 
     different chromosomes, then the probability of a certain event will
     be 3/16.  In an actual trial, the event occurs 56 times in 300.  
     Use Lindley's method to decide whether there is enough evidence to 
     reject the hypothesis that the genes are on the same chromosome.

\nextq With the data in the example in Section 3.4 on
     ``The Poisson distribution'', would it be appropriate to reject the
     hypothesis that the true mean equalled the prior mean (that is,
     that $\lambda=3$).  [Use Lindley's method.]
     
\nextq Suppose that the standard test statistic 
     $z=(\mean x-\theta_0)/\sqrt{(\phi/n)}$ takes the value $z = 2.5$
     and that the sample size is $n = 100$.  How close to $\theta_0$ 
     does a value of $\theta$ have to be for the value of the normal 
     likelihood function at $\mean x$ to be within 10\% of its value 
     at $\theta=\theta_0$?

\nextq Show that the Bayes factor for a test of a point null hypothesis
     for the normal distribution (where the prior under the alternative
     hypothesis is also normal) can be expanded in a power series in
     $\lambda=\phi/n\psi$ as
\[   B = \lambda^{-\frac{1}{2}}\exp(-\half z^2)\{1+\half\lambda(z^2+1)
         +\dots\}. \]

\nextq Suppose that $x_1$, $x_2$, $\dots$, $x_n\sim\N(0,\phi)$.  Show
     over the interval $(\phi-\varepsilon,\,\phi+\varepsilon)$ the
     likelihood varies by a factor of approximately
\[   \exp\left\{
     \frac{\varepsilon}{\phi}\left(\frac{\sum
     x_i^2/n}{\phi}-1\right)\right\}.\]

\nextq At the beginning of Section 4.5, we saw that 
     under the alternative hypothesis that $\theta \sim \N(\theta_0,
     \psi)$ the predictive density for $\mean x$ was 
     $\N(\theta_0, \psi+\phi/n)$, so that
\[   p_1(\mean x)=\{2\pi(\psi+\phi/n)\}^{-\frac{1}{2}}
                 \exp [-\half(\mean x - \theta_0)^2/(\psi+\phi/n)]      
     \]
     Show that a maximum of this density considered as a function of
     $\psi$ occurs when $\psi = (z^2 - 1)\phi/n$, which gives a 
     possible value for $\psi$ if $z \geqslant 1$.  Hence show that 
     if $z \geqslant 1$ then for any such alternative hypothesis the 
     Bayes factor satisfies
\[    B  \geqslant \sqrt{\text{e}}\, z \exp (-\half z^2)            \]
     and deduce a bound for $p_0$ (depending on the value of $\pi_0$).

\nextq In the situation discussed in Section 4.5,
     for a given $P$-value (so equivalently for a given $z$) and 
     assuming that $\phi$=$\psi$, at what value of $n$ is the 
     posterior probability of the null hypothesis a minimum.

\nextq Mendel
     (1865) reported finding 1850 angular wrinkled seeds to 5474 
     round or roundish in an experiment in which his theory predicted a 
     ratio of $1:3$.  Use the method employed for Weldon's dice data in 
     Section 4.5 to test whether his theory is 
     confirmed by the data.  [However, Fisher (1936) cast some doubt on 
     the genuineness of the data.]

\nextq A window 
     is broken in forcing entry to a house.  The refractive index 
     of a piece of glass found at the scene of the crime is $x$, which
     is supposed $\N(\theta_1, \phi)$.  The refractive index of a piece 
     of glass found on a suspect is $y$, which is supposed $N(\theta_2,
     \phi)$. In the process of establishing the guilt or innocence of the
     suspect, we are interested in investigating whether 
     $\Hyp_0: \theta_1 = \theta_2$ 
     is true or not.  The prior distributions of $\theta_1$ and
     $\theta_2$ 
     are both $N(\mu, \psi)$ where $\psi\gg\phi$.  Write
\[               u  =  x - y,\qquad      z  =  \half(x + y).           
\]
     Show that, if $\Hyp_0$ is true and $\theta_1 = \theta_2 = \theta$, 
     then $\theta$, $x - \theta$ and $y - \theta$ are independent and
\[    \theta \sim \N(\mu, \psi),\qquad x - \theta \sim \N(0, \phi),
      \qquad y - \theta \sim \N(0, \phi).      
\]
     By writing $u=(x-\theta)-(y-\theta)$ and 
     $z =\theta+ \half(x-\theta)+\half(y-\theta)$, go on to show that
     $u$ has an $\N(0, 2\phi)$ distribution and that $z$ has an 
     $\N(\mu,\half\phi+\psi)$, so approximately an $\N(\mu, \psi)$, 
     distribution.  Conversely, show that if $\Hyp_0$ is false and 
     $\theta_1$ and $\theta_2$ are assumed independent, then 
     $\theta_1$, $\theta_2$, $x - \theta_1$ and $y - \theta_2$ are all 
     independent and
\[     \theta_1\sim\N(\mu,\psi),\quad\theta_2\sim\N(\mu,\psi),\quad
       x - \theta_1\sim\N(0, \phi),\quad y - \theta_2 \sim \N(0, \phi).
\]
     By writing
\begin{align*}
     u &= \theta_1 - \theta_2 + (x - \theta_1) - (y - \theta_2),       
\\
     z &= \half\{\theta_1+\theta_2+(x-\theta_1)+(y-\theta_2)\}
\end{align*}
     show that in this case $u$ has an $\N(0, 2(\phi+\psi))$, so
     approximately an $\N(0,2\psi)$, distribution, while $z$ has an 
     $\N(\mu,\half(\phi+ \psi))$, so approximately an
     $\N(\mu,\half\psi)$, 
     distribution.  Conclude that the Bayes factor is approximately
\[    B=\sqrt{(\psi/2\phi)}\exp[-\half u^2/2\phi+\half(z-\mu)^2/\psi]. 
\]
     Suppose that the ratio $\sqrt{(\psi/\phi)}$ of the standard
     deviations is 100 and that $u = 2\times\sqrt{(2\phi)}$, so 
     that the difference between $x$ and $y$ represents two 
     standard deviations, and that 
     $z = \mu$, so that both specimens are of commonly occurring glass. 

     Show that a classical test would reject $\Hyp_0$ at the 5\% level, 
     but that $B = 9.57$, so that the odds in favour of $\Hyp_0$ are
     multiplied by a factor just below 10.

\blankline
     [This problem is due to Lindley 
     (1977); see also Shafer (1982).  Lindley 
     comments that, ``What the [classical] test fails to take into
     account is the extraordinary coincidence of $x$ and $y$ being so 
     close together were the two pieces of glass truly different''.]

\nextq Lindley (1957) originally discussed his paradox 
     under slightly different assumptions from those made in this 
     book.  Follow through the reasoning 
     used in Section 4.5 with $\rho_1(\theta)$ 
     representing a uniform distribution 
     on the interval $(\theta_0-\half\tau,\,\theta_0+\half\tau)$ 
     to find the corresponding Bayes factor assuming that
     $\tau^2\gg\phi/n$, 
     so that an $\N(\mu, \phi/n)$ variable lies in this interval with
     very high probability.  Check that your answers are unlikely to 
     disagree with those found in  Section 4.5 under the 
     assumption that $\rho_1(\theta)$ represents a normal density.

\nextq Express in your own words the arguments given by Jeffreys (1961, 
     Section 5.2) in favour of a Cauchy distribution
\[  
\rho_1(\theta)=\frac{1}{\pi}\frac{\sqrt{\psi}}{\psi+(\theta-\theta_0)^2}
\]
     in the problem discussed in the previous question.

\nextq Suppose that $x$ has a binomial distribution 
     $B(n, \theta)$ of index 
     $n$ and parameter $\theta$, and that it is desired to test 
     $\Hyp_0: \theta = \theta_0$ against the alternative hypothesis 
     $\Hyp_1: \theta\neq\theta_0$.

\begin{description}
\item[\quad(a)]  Find lower bounds on the posterior probability of
$\Hyp_0$ and 
     on the Bayes factor for $\Hyp_0$ versus $\Hyp_1$, bounds which are
valid 
     for any $\rho_1(\theta)$.

\item[\quad(b)]  If $n = 20$, $\theta_0 = \frac{1}{2}$ and $x = 15$ is 
     observed, calculate the (two-tailed) $P$-value and the lower bound 
     on the posterior probability when the prior probability $\pi_0$ of 
     the null hypothesis is $\half$.
\end{description}

\nextq Twelve observations from a normal distribution of mean $\theta$
     and variance $\phi$ are available, of which the sample mean is 1.2 
     and the sample variance is 1.1.  Compare the Bayes factors in 
     favour of the null hypothesis that $\theta=\theta_0$ assuming 
     (a) that $\phi$ is unknown and (b) that it is known that $\phi = 1$.

\nextq Suppose that in testing a point null hypothesis you find a value
     of the usual Student's $\t$ 
     statistic of 2.4 on 8 degrees of freedom.  Would the methodology of
     Section 4.6 require you to ``think again''?

\nextq Which entries in the table in Section 4.5
     on ``Point null hypotheses for the normal distribution would,
     according to the methodology of Section 4.6, cause you to
     ``think again''?

\section{Exercises on Chapter \arabic{section}}

\setcounter{qno}{0}

\nextq Two analysts measure the percentage of ammonia in a chemical
     process over 9 days and find the following discrepancies 
     between their results:
\[
\begin{array}{lccccccccc}
\text{Day}      &  1  &  2  &  3  &  4  &  5  &  6  &  7  &  8  &  9   
\\
\text{Analyst A}&12.04&12.37&12.35&12.43&12.34&12.36&12.48&12.33&12.33 
\\
\text{Analyst B}&12.18&12.37&12.38&12.36&12.47&12.48&12.57&12.28&12.42
\end{array}
\]
     Investigate the mean discrepancy $\theta$ between their results 
     and in particular give an interval in which you are 90\% sure 
     that it lies.

\nextq With the same data as in the previous question, test the
     hypothesis that there is no discrepancy between the two analysts.

\nextq Suppose that you have grounds for believing that observations
     $x_i$, $y_i$ for $i=1$, 2, \dots, $n$ are such that
     $x_i\sim\N(\theta,\phi_i)$ and also $y_i\sim\N(\theta,\phi_i)$, but
     that you are not prepared to assume that the $\phi_i$ are equal.
     What statistic would you expect to base inferences about $\theta$
     on?

\nextq How much difference would it make to the analysis of the data in
     Section 5.1 on rat diet if we took
     $\omega=\half(\phi+\psi)$ instead of $\omega=\phi+\psi$.

\nextq Two analysts in the same laboratory made repeated determinations
     of the percentage of fibre in soya cotton cake, the results being 
     as shown below:
\[
\begin{array}{llllllllll}
\text{Analyst A}&12.38&12.53&12.25&12.37&12.48&12.58&12.43&12.43&12.30
\\
\text{Analyst B}&12.25&12.45&12.31&12.31&12.30&12.20&12.25&12.25&12.26
\\
                &12.42&12.17&12.09
\end{array}
\]
     Investigate the mean discrepancy $\theta$ between their mean 
     determinations and in particular give an interval in which you are 
     90\% sure that it lies
\begin{description}
\item[\quad(a)] assuming that it is known from past experience that the 
     standard deviation of both sets of observations is 0.1, and

\item[\quad(b)] assuming simply that it is known that the standard
     deviations of the two sets of observations are equal.
\end{description}

\nextq A random sample $\vect x = (x_1, x_2, \dots, x_m)$ is available 
     from an $\N(\lambda,\phi)$ distribution and a second independent
     random sample $\vect y = (y_1, y_2, \dots, y_n)$ is available 
     from an $\N(\mu, 2\phi)$ distribution.  Obtain, under the usual
     assumptions, the posterior distributions of $\lambda-\mu$ and of 
     $\phi$.

\nextq Verify the formula for $S_1$ given towards the end of Section 
     5.2.

\nextq The following data consists of the lengths in mm of cuckoo's eggs
     found in nests belonging to the dunnock and to the reed warbler:
\[
\begin{array}{lllllllllll}
\text{Dunnock}      &22.0&23.9&20.9&23.8&25.0&24.0&21.7&23.8&22.8&23.1
\\
\text{Reed warbler} &23.2&22.0&22.2&21.2&21.6&21.9&22.0&22.9&22.8
\end{array}
\]
     Investigate the difference $\theta$ between these lengths without 
     making any particular assumptions about the variances of the two 
     populations, and in particular give an interval in which you are 
     90\% sure that it lies.

\nextq Show that if $m=n$ then the expression $f_1^2/f_2$ in Patil's
     approximation reduces to
\[     \frac{4(m-5)}{3 + \cos 4\theta}. \]

\nextq Suppose that $T_x$, $T_y$ and $\theta$ are defined as in Section 
     5.3 
     and that
\[    T=T_x\sin\theta-T_y\cos\theta,\qquad
      U=T_x\cos\theta+T_y\sin\theta \]
     Show that the transformation from $(T_x, T_y)$ to $(T, U)$ has unit
     Jacobian and hence show that the density of $T$ satisfies
\begin{align*}
     p(T|\vect x,\vect y) &\propto \int_0^{\infty}
          [1  +  (T\sin\theta+U\cos\theta)^2/\nu_x]^{-(\nu(x)+1)/2}    
\\
     &\qquad\times[1  + 
(-T\cos\theta+U\sin\theta)^2/\nu_y]^{-(\nu(y)+1)/2}\dU.
\end{align*}

\nextq Show that if $x\sim\F_{\nu_1,\nu_2}$ then
\[     \frac{\nu_1x}{\nu_2+\nu_1x} \sim \Be(\half\nu_1,\half\nu_2). \]

\nextq Two different microscopic methods, $A$ and $B$, are available for
     the measurement of very small dimensions in microns.  As a result of 
     several such measurements on the same object, estimates of variance
     are available as follows:
\[
\begin{array}{lll}
          \text{Method}\hspace{25mm}    &       A\hspace{20mm}         
&       B       \\
          \text{No. of observations}    &     m = 15       &     n = 25 
  \\
          \text{Estimated variance}     &   s_1^2 = 7.533  &   s_2^2 =
1.112
\end{array}
\]
     \,\,Give an interval in which you are 95\% sure that the ratio
     of the variances lies.

\nextq Measurement errors when using two different instruments are
     more or less symmetrically distributed and are believed to be
     reasonably well approximated by a normal distribution.  Ten
     measurements with each show a sample standard deviation three times
     as large with one instrument as with the other.  Give an interval
     in which you are 99\% sure that the ratio of the true standard
     deviations lies.

\nextq Repeat the analysis of Di Raimondo's data in Section 
     5.6 on the effects of penicillin of mice, this 
     time assuming that you have prior knowledge worth about six
     observations in each case suggesting that the mean chance of
     survival is about a half with the standard injection but about
     two-thirds with the penicillin injection.
     
\nextq The table below [quoted from Jeffreys (1961, Section 5.1)] 
     gives the relationship between grammatical gender in Welsh and 
     psychoanalytical symbolism according to Freud:
\[
\begin{array}{lll}
  \text{Psycho. $\backslash$ Gram.}\hspace{5mm} & M\hspace{10mm} &  F   
  \\
           M                                    &   45           &    
30  \\
           F                                    &   28           &    
29  \\
       \text{Total}                             &   73           &    
59
\end{array}
\]
     Find the posterior probability that the log odds-ratio is positive 
     and compare it with the comparable probability found by using the 
     inverse root-sine transformation.

\nextq Show that if $\pi \cong \rho$ then the log odds-ratio is such
     that
\[            \Lambda-\Lambda' \cong (\pi-\rho)/\{\pi(1 - \pi)\}.     
\]

\nextq A report issued in 1966 about the effect of radiation on patients
     with inoperable lung cancer compared the effect of radiation
     treatment with placebos.  The numbers surviving after a year were:
{\catcode`?=\active
 \def?{\kern\digitwidth}
\[
\begin{array}{lll}
  \hspace{25mm}        & \text{Radiation}\hspace{5mm} & \text{Placebos} 
\\
  \text{No. of cases}  &      308                     &        246      
\\
  \text{No. surviving} &      ?56                     &        ?34
\end{array}
\]
}
     \!\!What are the approximate posterior odds that the one-year
     survival rate of irradiated patients is at least 0.01 greater than 
     that of those who were not irradiated?

\nextq Suppose that $x \sim \P(8.5)$, i.e. $x$ is Poisson 
     of mean 8.5, 
     and $y \sim \P(11.0)$.  What is the approximate distribution of 
     $x - y$?

\section{Exercises on Chapter \arabic{section}}

\setcounter{qno}{0}

\nextq The sample correlation coefficient 
     between length and weight of a 
     species of frog was determined at each of a number of sites.  The 
     results were as follows:
{\catcode`?=\active
 \def?{\kern\digitwidth}
\[
\begin{array}{llllll}
\text{Site}&              ?1&      ?2&      ?3&      ?4&      ?5      
\\   
\text{Number of frogs}&   12&      45&      23&      19&      30      
\\
\text{Correlation}&       ?0.631&  ?0.712&  ?0.445&  ?0.696&  ?0.535
\end{array}
\]
}
     Find an interval in which you are 95\% sure that the correlation 
     coefficient lies.

\nextq Three groups of children were given two tests.  The numbers of 
     children and the sample correlation coefficients 
     between the two test 
     scores in each group were as follows:
\[
\begin{array}{lccc}
     \text{Number of children}&   45&     34&      49     \\
     \text{Correlation}&        0.489&   0.545&   0.601
\end{array}
\]
     Is there any evidence that the association between the two tests 
     differs in the three groups?

\nextq Suppose you have sample correlation coefficients $r_1$, $r_2$,
     $\dots$, $r_k$ on the basis of sample sizes $n_1$, $n_2$, $\dots$,
     $n_k$.  Give a 95\% posterior confidence interval for
     $\zeta=\tanh^{-1}\rho$.

\nextq From the approximation
\[   p(\rho|\vect x,\vect y) \propto (1 - \rho^2)^{n/2}(1 - \rho r)^{-n}
\]
     which holds for large $n$, deduce an expression for the
     log-likelihood $L(\rho|\vect x,\vect y)$ and hence show that 
     the maximum likelihood occurs when $\rho = r$.  An approximation 
     to the information can now be 
     made by replacing $r$ by $\rho$ in the second derivative of the 
     likelihood, since $\rho$ is near $r$ with high probability.  Show
     that this approximation suggests a prior density of the form
\[          p(\rho)  \propto  (1 - \rho^2)^{-1}.       \]

\nextq Use the fact that
\[   \int_0^{\infty} (\cosh t+\cos\theta)^{-1}\dt=\theta/\sin\theta  \]
     (cf.\ Edwards, 1921, art.\ 180) to show that
\[   p(\rho|\vect x,\vect y)  \propto  p(\rho) (1 - \rho^2)^{(n-1)/2}
       \frac{d^{n-2}}{\d(\rho r)^{n-2}}
       \left(\frac{\arccos(-\rho r)}{\sqrt{(1-\rho^2r^2)}}\right). \]

\nextq Show that in the special case where the sample correlation 
     coefficient 
     $r = 0$ and the prior takes the special form 
     $p(\rho) \propto (1 - \rho^2)^k$ the variable
\[                \sqrt{(k + n + 1)} \rho/(1 - \rho^2)             \]
     has a Student's $\t$ distribution on $k + n + 1$ degrees of
     freedom.

\nextq By writing
\begin{align*}
     \omega^{-1}(\omega + \omega^{-1} - 2\rho r)^{-(n-1)}
     &= \omega^{n-2}(1 - \rho^2)^{-(n-1)}                            \\
     &\qquad\times [1+(\omega-\rho r)^2 (1-\rho^2r^2)^{-1}]^{-(n-1)}
\end{align*}
     and using repeated integration by parts, show that the posterior 
     distribution of $\rho$ can be expressed as a finite series
     involving powers of
\[                \sqrt{(1 - \rho r)/(1 + \rho r)}                   \]
     and Student's $\t$ integrals.

\nextq By substituting
\[    \cosh t - \rho r = \frac{1-\rho r}{1-u}                        \]
     in the form
\[    p(\rho|\vect x,\vect y) \propto p(\rho) (1 - \rho^2)^{(n-1)/2}
      \int_0^{\infty} (\cosh t  -  \rho r)^{-(n-1)} \dt               
\]
     for the posterior density of the correlation coefficient 
     and then expanding
\[                          [1  -  \half(1 + \rho r)u]^{-\frac{1}{2}} 
\]
     as a power series in u, show that the integral can be expressed as
     a series of beta functions.  Hence deduce that
\[   p(\rho|\vect x,\vect y) \propto p(\rho) (1 - \rho^2)^{(n-1)/2}
     (1-\rho r)^{-n+(3/2)}S_n(\rho r)                                  
\]
     where
\[   S_n(\rho r)=1 + \sum_{l=1}^{\infty} \frac{1}{l!}
     \left(\frac{1+\rho r}{8}\right)^l
     \prod_{s=1}^l \frac{(2 s-1)^2}{(n-\frac{3}{2}+s)}.                 
\]

\nextq  Fill in the details of the derivation of the prior
\[   p(\phi,\psi,\rho) \propto (\phi\psi)^{-1} (1 - \rho^2)^{-3/2}     
\]
     from Jeffreys' rule 
     as outlined at the end of Section 6.1.

\nextq The data below consist of the estimated gestational ages (in 
     weeks) and weights (in grammes) of twelve female babies:
{
\[
\begin{array}{lc@{}c@{}c@{}c@{}c@{}c@{}c@{}c@{}c@{}c@{}c@{}c}
\text{Age}&    40 & 36 & 40 & 38 & 42 & 39 & 40 & 37 & 36 & 38 & 39 & 40
 \\
\text{Weight}&\,3317\,&\,2729\,&\,2935\,&\,2754\,&\,3210\,&\,2817\,&
              \,3126\,&\,2539\,&\,2412\,&\,2991\,&\,2875\,&\,3231\,
\end{array}
\]
}
     Give an interval in which you are 90\% sure that the gestational
     age of a particular such baby will lie if its weight is 3000 
     grammes, and give a similar interval in which the mean weight of 
     all such babies lies.

\nextq Show directly from the definitions that, in the notation of
     Section 6.3,
\[        S_{ee} = \sum\{y_i  -  a  -  b(x_i - \mean x)\}^2.          \]

\nextq Observations $y_i$ for $i = -m, -m + 1, \dots, m$ are available 
     which satisfy the regression 
     model 
\[          y_i  \sim  \N(\alpha + \beta u_i + \gamma v_i,\,\phi)     \]
     where $u_i = i$ and $v_i = i^2 - \half m(m + 1)$.  Adopting the 
     standard reference prior 
     $p(\alpha, \beta, \gamma, \phi)$ $\propto 1/\phi$, 
     show that the posterior distribution of $\alpha$ is such that
\[   \frac{\alpha-\mean y}{s/\sqrt{n}}\sim\t_{n-3}                    \]
     where $n = 2 m + 1$, $s^2 = S_{ee}/(n - 3)$ and
\[   S_{ee}  =  S_{yy}  -  S_{uy}^2/S_{uu}  -  S_{vy}^2/S_{vv}        \]
     in which $S_{yy}$, $S_{uy}$, etc., are defined by
\[   S_{yy}  =  \sum (y_i - \mean y)^2,\qquad 
     S_{uy}  =  \sum (u_i - \mean u))(y_i - \mean y).                 \]
     [\textit{Hint:} Note that $\sum u_i = \sum v_i = \sum u_iv_i = 0$,
     and hence $\mean u=\mean v=0$ and $S_{uy}=0$.]

\nextq Fisher (1925b, Section 41) quotes an experiment on the accuracy
     of counting soil bacteria.  In it, a soil sample was divided into 
     four parallel samples, and from each of theses after dilution seven
     plates were inoculated.  The number of colonies on each plate is 
     shown below.  Do the results from the four samples agree within 
     the limits of random sampling?
\[
\begin{array}{lcccc}
     \text{Plate $\backslash$ Sample}\hspace{5mm} &
     \hspace{5mm}\text{A}\hspace{5mm}&
     \hspace{5mm}\text{B}\hspace{5mm}&
     \hspace{5mm}\text{C}\hspace{5mm}&
     \hspace{5mm}\text{D}\hspace{5mm}                                   
  \\
       \quad 1                        &    72  &    74  &    78  &    69
  \\
       \quad 2                        &    69  &    72  &    74  &    67
  \\
       \quad 3                        &    63  &    70  &    70  &    66
  \\
       \quad 4                        &    59  &    69  &    58  &    64
  \\
       \quad 5                        &    59  &    66  &    58  &    64
  \\
       \quad 6                        &    53  &    58  &    56  &    58
  \\
       \quad 7                        &    51  &    52  &    56  &    54
\end{array}
\]

\nextq In the case of the data on scab disease quoted in Section 
     6.5, find a contrast measuring the effect of the
     season in which sulphur is applied and give an appropriate HDR for
     this contrast.

\nextq The data below [from Wishart and Sanders (1955, Table 5.6)] 
     represent the weight of green produce in pounds made on an 
     old pasture.  There were three main treatments, including a 
     control (O) consisting of the untreated land.  In the
     other cases the effect of a grass-land rejuvenator (R) was
     compared with the use of the harrow (H).  The blocks were
     therefore composed of three plots each, and the experiment
     consisted of six randomized blocks placed side by side.  The 
     plan and yields were as follows:

\vspace{0.75\baselineskip}
{\small
  \hbox{
    \hspace{-1.2em}
    \begin{tabular}{c@{\ }c@{\ }c|c@{\ }c@{\ }c|c@{\ }c@{\ }c|
                    c@{\ }c@{\ }c|c@{\ }c@{\ }c|c@{\ }c@{\ }c}
       O.& H.& R.& R.& H.& O.& O.& R.& H.& O.& R.& H.& H.& O.& R.& O.&
H.& R.\\
813&647&713&814&759&795&705&652&598&774&617&559&580&687&539&581&480&437
    \end{tabular}
  }
}
\vspace{0.75\baselineskip}

\noindent
Derive an appropriate two-way analysis of variance.
     
\nextq Express the two-way layout as a particular case of the general
     linear model.

\nextq Show that the matrix 
     $\matr A^{+}=(\matr A\transpose\matr A)^{-1}\matr A\transpose$
     which arises in the theory of the general linear model is a
     \textit{generalized inverse} of the (usually non-square) matrix
     $\matr A$ in that
\begin{description}
  \item[\quad(a)] $\matr A\matr A^{+}\matr A=\matr A$
  \item[\quad(b)] $\matr A^{+}\matr A\matr A^{+}=\matr A^{+}$
  \item[\quad(c)] $(\matr A\matr A^{+})\transpose=\matr A\matr A^{+}$
  \item[\quad(d)] $(\matr A^{+}\matr A)\transpose=\matr A^{+}\matr A$
\end{description}

\nextq Express the bivariate linear regression model in terms of the
     original parameters $\beeta=(\eta_0,\eta_1)\transpose$ and the
     matrix $\matr A_0$ and use the general linear model to find the
     posterior distribution of $\beeta$.

\section{Exercises on Chapter \arabic{section}}

\setcounter{qno}{0}

\nextq Show that in any experiment $E$ in which there is a possible 
     value $y$ for the random variable $\random x$ such that 
     $p_{\random x}(y|\theta) = 0$, then if $z$ is any other possible
value 
     of $\random x$, the statistic $t = t(x)$ defined by
\[   t(x) = \left\{\begin{array}{ll}
                    z  &  \text{if $x=y$}          \\
                    x  &  \text{if $x\neq y$}
                    \end{array}\right.             \]
     is sufficient 
     for $\theta$ given $x$.  Hence show that if $\random x$ is a 
     continuous random variable, then a na\"\i ve application of the 
     weak sufficiency principle as defined in Section 7.1
     would result in $\Ev\{E, y, \theta\} = \Ev\{E, z, \theta\}$ for any
     two possible values $y$ and $z$ of $\random x$.

\nextq Consider an experiment $E = \{\random x,\theta, p(x|\theta)\}$.
     We say that \textit{censoring} (strictly speaking, fixed censoring)
     occurs with censoring mechanism $g$ (a known function of $x$) when,
     instead of $\random x$, one observes $y=g(x)$.  A typical example 
     occurs when we report $x$ if $x < k$ for some fixed $k$, but
     otherwise simply report that $x\geqslant k$.  As a result, the 
     experiment really performed is 
     $E^g = \{\random y,\theta, p(y|\theta)\}$.  
     A second method with censoring mechanism $h$ is said to be 
     \textit{equivalent} to the first when
\[ g(x)=g(x')\text{\quad if and only if\quad}h(x)=h(x'). \]
     As a special case, if $g$ is one-to-one then the mechanism is said
     to be equivalent to no censoring.  Show that if two censoring 
     mechanisms are equivalent, then the likelihood principle implies 
     that 
\[ \Ev\{E^g, x, \theta\}=\Ev\{E^h, x, \theta\}. \]

\nextq Suppose that the density function $p(x|\theta)$ is defined as 
     follows for $x = 1, 2, 3, \dots$ and $\theta = 1, 2, 3, \dots$.  
     If $\theta$ is even, then
\begin{align*}
     p(x|\theta)&=\left\{\begin{array}{ll}
                        \frac{1}{3} & \text{if $x=\theta/2$, $2\theta$
                                             or $2\theta+1$}            
  \\
                        0           & \text{otherwise}
                         \end{array}\right. \\
\intertext{if $\theta$ is odd but $\theta\neq1$, then}
     p(x|\theta)&=\left\{\begin{array}{ll}
                        \frac{1}{3} & \text{if $x=(\theta-1)/2$,
$2\theta$
                                             or $2\theta+1$}            
   \\
                        0           & \text{otherwise}
                         \end{array}\right. \\
\intertext{while if $\theta = 1$ then}
     p(x|\theta)&=\left\{\begin{array}{ll}
                        \frac{1}{3} & \text{if $x=\theta$, $2\theta$
                                            or $2\theta+1$}             
   \\
                        0     & \text{otherwise}
                         \end{array}\right.
\end{align*}
     Show that, for any $x$ the data intuitively give equal support 
     to the three possible values of $\theta$ compatible with that 
     observation, and hence that on likelihood grounds any of the three 
     would be a suitable estimate.  Consider, therefore, the three 
     possible estimators $d_1$, $d_2$ and $d_3$ corresponding to the 
     smallest, middle and largest possible $\theta$.  Show that
\begin{align*}
       p(d_2=1)&=\left\{\begin{array}{ll}
                       \frac{1}{3} & \text{when $\theta$ is even}  \\
                       0           & \text{otherwise}
                        \end{array}\right. \\
       \ \\
       p(d_3=1)&=\left\{\begin{array}{ll}
                       \frac{1}{3} & 
                            \text{when $\theta$ is odd but
$\theta\neq1$}  \\
                       0           & \text{otherwise}
                        \end{array}\right. \\
\intertext{but that}
       p(d_1=1)&=\left\{\begin{array}{ll}
                       1           & \text{when $\theta=1$}         \\
                       \frac{2}{3} & \text{otherwise}
                        \end{array}\right.
\end{align*}
     Does this apparent discrepancy cause any problems for a Bayesian 
     analysis? (due to G.~Monette and D.~A.~S.~Fraser).

\nextq A drunken soldier, 
     starting at an intersection O in a city which 
     has square blocks, staggers around a random path trailing a taut 
     string.  Eventually he stops at an intersection (after walking 
     at least one block) and buries a treasure.  Let $\theta$ denote 
     the path of the string from O to the treasure.  Letting $N$, $S$, 
     $E$ and $W$ stand for a path segment one block long in the
     indicated direction, so that $\theta$ can be expressed as a 
     sequence of such letters, say $\theta =\!\!\textit{NNESWSWW}$.  
     (Note that $NS$, $SN$, $EW$ and $WE$ cannot appear as the taut 
     string would be rewound).  
     After burying the treasure, the soldier walks one block further in 
     a random direction (still keeping the string taut).  Let $X$ denote
     this augmented path, so that $X$ is one of $\theta N$, $\theta S$, 
     $\theta E$ and $\theta W$, each with probability $\quarter$.  You 
     observe $X$ and are then to find the treasure.  Show that if you 
     use a reference 
     prior $p(\theta) \propto 1$ for all possible paths 
     $\theta$, then all four possible values of $\theta$ given $X$ are 
     equally likely.  Note, however, that intuition would suggest that 
     $\theta$ is three times as likely to extend the path as to
     backtrack, suggesting that one particular value of $\theta$ is 
     more likely than 
     the others after $X$ is observed.  (Due to M. Stone).

\nextq Suppose that, starting with a fortune of $f_0$ units, you bet $a$
     units each time on evens at roulette (so that you have a 
     probability of 18/37 of winning at Monte Carlo or 18/38 at Las 
     Vegas) and keep a record of your fortune $f_n$ and the difference 
     $d_n$ between the number of times you win and the number of times 
     you lose in $n$ games.  Which of the following are stopping times?
\begin{description}
  \item[\quad(a)] The last time $n$ at which $f_n\geqslant f_0$?
  \item[\quad(b)] The first time that you win in three successive games?
  \item[\quad(c)] The value of $n$ for which 
                  $f_n=\max_{\,\{0\leqslant k < \infty\}} f_k$ ? 
\end{description}

\nextq Suppose that $x_1, x_2, \dots$ is a sequential sample from an 
     $\N(\theta, 1)$ distribution and it is desired to test 
     $\Hyp_0: \theta = \theta_0$ versus $\Hyp_1: \theta\neq\theta_0$.  
     The experimenter reports that he used a proper stopping rule 
     and obtained the data 3, $-1$, 2, 1.
\begin{description}
\item[(a)] What could a frequentist conclude?

\item[(b)] What could a Bayesian conclude?
\end{description}

\nextq Let $x_1, x_2, \dots$ be a sequential sample from a Poisson 
     distribution 
     $\P(\lambda$).  Suppose that the stopping rule is to 
     stop sampling at time $n \geqslant 2$ with probability
\[    \sum_{i=1}^{n-1} x_i \left/ \sum_{i=1}^n x_i\right.           \]
     for $n = 2, 3, \dots$ (define $0/0 = 1$).  Suppose that the first 
     five observations are 3, 1, 2, 5, 7 and that sampling then stops.  
     Find the likelihood function for $\lambda$.  (Berger, 1985).

\nextq Show that the mean of the beta-Pascal distribution
\[     p(S|R, r, s)=\binom{S}{s}\frac{\Betafn(r''+s, R''-r''+S-s)}
                                     {\Betafn(r''-1, R''-r'')}       \]
     is given by the formula in Section 7.3, namely,
\[    \E S=(s+1)\left(\frac{R''-2}{r''-2}\right)-1                  \]

\nextq Suppose that you intend to observe the number $x$ of successes 
     in $n$ Bernoulli trials 
     and the number $y$ of failures before the 
     $n$th success after the first $n$ trials, so that $x\sim\B(n,\pi)$ 
     and $y\sim\NB(n,\pi)$.  
     Find the likelihood function $L(\pi|x, y)$ 
     and deduce the reference prior 
     that Jeffreys' rule 
     would suggest for this case.

\nextq The negative of loss is sometimes referred to as
     \textit{utility}.  Consider a gambling game very unlike most in
     that you are bound to win at least $\pounds 2$, and accordingly in
     order to be allowed to play, you must pay an entry fee of 
     $\pounds e$.  
     A coin is tossed until it comes up heads, and if this occurs 
     for the first time on the $n$th toss, you receive $\pounds 2^n$.  
     Assuming that the utility to you of making a gain of $\pounds x$ is
     $u(x)$, find the expected utility of this game, and then discuss
     whether it is plausible that $u(x)$ is directly proportional to
     $x$. [The gamble discussed here is known as the \textit{St
     Petersburg Paradox}.  A fuller discussion of it can be found in 
     Leonard and Hsu (1999, Chapter 4).]

\nextq Suppose that you want to estimate the parameter $\pi$ of a
binomial 
     distribution $\B(n, \pi)$.  Show that if the loss function is
\[       L(\theta, a)  =  (\theta - a)^2/\{\theta(1 - \theta)\}  \]
     then the Bayes rule corresponding to a uniform (that is,
     $\Be(1,1)$) prior for $\pi$ is given by $d(x) = x/n$ for any 
     $x$ such that $0 < x < n$, that is, the maximum likelihood 
     estimator.  Is $d(x) = x/n$ 
     a Bayes rule if $x = 0$ or $x = n$?

\nextq Let $x\sim\B(n, \pi)$ and $y\sim\B(n, \rho)$ have independent 
     binomial distributions 
     of the same index but possibly different 
     parameters.  Find the Bayes rule 
     corresponding to the loss
\[      L((\pi,\rho), a)   =  (\pi - \rho - a)^2                 \]
     when the priors for $\pi$ and $\rho$ are independent uniform 
     distributions.

\nextq Investigate possible point estimators for $\pi$ on the
     basis of the posterior distribution in the example in
     the subsection of Section 2.10 headed
     ``Mixtures of conjugate densities''.

\nextq Find the Bayes rule corresponding to the loss function
\[     L(\theta, a)=\left\{\begin{array}{ll}
                            u(a-\theta) & \mbox{if $a\leqslant\theta$}
\\
                            v(\theta-a) & \mbox{if $a\geqslant\theta$}.
                          \end{array}\right. \]

\nextq Suppose that your prior for the proportion $\pi$ of defective 
     items supplied by a manufacturer is given by the beta distribution 
     $\Be(2, 12)$, and that you then observe that none of a random
     sample of size 6 is defective.  Find the posterior distribution 
     and use it to carry out a test of the hypothesis 
     $\Hyp_0: \pi < 0.1$ using
\begin{description}
\item[(a)] a ``0 -- 1'' loss function, and

\item[(b)] the loss function
\[
\hspace{-0.9cm}
\begin{array}{lll}
  a\backslash\theta\hspace{10mm} & \theta\in\Theta_0\hspace{10mm} &  
                                   \theta\in\Theta_1  \\
    a_0                &    0                  &  1                  \\
    a_1                &    2                  &  0
\end{array}
\]
\end{description}

\nextq Suppose there is a loss function $L(\theta, a)$ defined by
\[
{\catcode`?=\active
  \def?{\kern\digitwidth}
\begin{array}{lll}
  a\backslash\theta\hspace{10mm} & \theta\in\Theta_0\hspace{10mm} &  
                                   \theta\in\Theta_1  \\
    a_0                &    ?0                  &  10                 
\\
    a_1                &    10                  &  ?0                 
\\
    a_2                &    ?3                  &  ?3
\end{array}
}
\]
     On the basis of an observation $x$ you have to take action 
     $a_0$, $a_1$ or $a_2$.  For what values of the posterior 
     probabilities $p_0$ and $p_1$ of the hypotheses 
     $\Hyp_0: \theta\in\Theta_0$ and $\Hyp_1:
     \theta\in\Theta_1$ would you take each of the possible actions?

\nextq A child is given an intelligence test.  We assume that the test 
     result $x$ is $\N(\theta, 100)$ where $\theta$ is the true
     intelligence quotient of the child, as measured by the test 
     (in other words, if 
     the child took a large number of similar tests, the average score 
     would be $\theta$).  Assume also that, in the population as a
     whole, $\theta$ is distributed according to an $\N(100, 225)$
     distribution.  If it is desired, on the basis of the intelligence
     quotient, to decide whether to put the child into a slow, average 
     or fast group for reading, the actions available are:
     \blankline
     
          $a_1:$ Put in slow group, that is, decide 
                    $\theta\in\Theta_1 = (0, 90)$

          $a_1:$ Put in average group, that is, decide 
                    $\theta\in\Theta_2 = [90, 100]$

          $a_1:$ Put in fast group, that is, decide 
                    $\theta\in\Theta_3 = (100, \infty).$

\blankline
     A loss function $L(\theta, a)$ of the following form might be deemed 
     appropriate:
\[
\begin{array}{llll}
  a\backslash\theta\quad\ 
                     & \theta\in\Theta_1  & \theta\in\Theta_2  
                                          & \theta\in\Theta_3           
  \\
  a_1                & 0                  & \theta-90       &
2(\theta-90) \\
  a_2                & 90-\theta          & 0               & \theta-110
  \\
  a_3                & 2(110-\theta)\quad & 110-\theta\quad & 0
\end{array}
\]
     Assume that you observe that the test result $x = 115$.  By using 
     tables of the normal distribution and the fact that if $\phi(t)$ is
     the density function of the standard normal distribution, then 
     $\int t \phi(t) \dt = -\phi(t)$, find is the appropriate action to 
     take on the basis of this observation.  [See Berger (1985, 
     Sections 4.2, 4.3, 4.4)].

\nextq In Section 7.8, a point estimator 
     $\delta_n$ for the current value 
     $\lambda$ of the parameter of a Poisson distribution 
     was found.  Adapt 
     the argument to deal with the case where the underlying
     distribution is geometric, that is
\[    p(x|\pi) = \pi(1-\pi)^x.                             \]
     Generalize to the case of a negative binomial distribution, that
     is,
\[    p(x|\pi) = \binom{n+x-1}{x} \pi^n (1-\pi)^x.         \]

\section{Exercises on Chapter \arabic{section}}

\setcounter{qno}{0}

\nextq Show that the prior 
\[ p(\alpha,\beta) \propto (\alpha + \beta)^{-5/2} \]
     suggested in connection with the example on risk of tumour in a 
     group of rats is equivalent to a density uniform in
     $\left(\alpha/(\alpha+\beta),\,(\alpha+\beta)^{-1/2}\right)$.

\nextq Observations $x_1$, $x_2$, \dots, $x_n$ are independently
     distributed given parameters $\theta_1$, $\theta_2$, \dots, 
     $\theta_n$ according to the Poisson distribution
     $p(x_i|\btheta)=\theta_i^{x_i}\exp(-\theta_i)/x_i!$.  The prior
     distribution for $\btheta$ is constructed hierarchically.  First
     the $\theta_i$s are assumed to be independently identically 
     distributed
     given a hyperparameter $\phi$ according to the exponential 
     distribution $p(\theta_i|\phi)=\phi\exp(-\phi\theta_i)$ for 
     $\theta_i\geqslant 0$ and then $\phi$ is given the improper uniform
     prior $p(\phi)\propto 1$ for $\phi\geqslant 0$.  Provided that 
     $n\mean x>1$, prove that the posterior distribution of 
     $z=1/(1+\phi)$ has the beta form
\[ p(z|\vect x)\propto z^{n\mean x-2}(1-z)^n. \]
     Thereby show that the posterior means of the $\theta_i$ are shrunk 
     by a factor $(n\mean x-1)/(n\mean x+n)$ relative to the usual 
     classical procedure which estimates each of the $\theta_i$ by
     $x_i$.

     What happens if $n\mean x\leqslant 1$?

\nextq Carry out the Bayesian analysis for known overall mean developed 
     in Section 8.2 above (a) with the loss 
     function replaced by a weighted mean 
\[ L(\btheta,\est\btheta)=\sum_{i=1}^r w_i(\theta_i-\est\theta_i)^2, \]
     and (b) with it replaced by
\[ L(\btheta,\est\btheta)=\sum_{i=1}^r |\theta_i-\est\theta_i|. \]

\nextq Compare the effect of the Efron-Morris estimator on the baseball
     data in Section 8.3 with the effect of a James-Stein
     estimator which shrinks the values of $\pi_i$ towards 
     $\pi_0=0.25$ or equivalently shrinks the values of $X_i$ towards 
     $\mu=2\sqrt{n}\sin^{-1}\sqrt{\pi_0}$.

\nextq The \textit{Helmert transformation} is defined by the matrix
\[ \matr A=\left(\begin{array}{cccccc}
   r^{-1/2}&2^{-1/2} &6^{-1/2}&12^{-1/2}         &\dots
&\{r(r-1)\}^{-1/2} \\
   r^{-1/2}&-2^{-1/2}&6^{-1/2}&12^{-1/2}         &\dots
&\{r(r-1)\}^{-1/2} \\
   r^{-1/2}&0        &-2\times 6^{-1/2}&12^{-1/2}&\dots
&\{r(r-1)\}^{-1/2} \\
   r^{-1/2}&0        &0&-3\times 12^{-1/2}       &\dots
&\{r(r-1)\}^{-1/2} \\
   r^{-1/2}&0        &0       &0                 &\dots
&\{r(r-1)\}^{-1/2} \\
   \vdots  &\vdots   &\vdots  &\vdots            &\ddots&\vdots \\
   r^{-1/2}&0        &0       &0                 &\dots
&-(r-1)^{1/2}r^{-1/2} \\
   \end{array}\right)
\]
     so that the element $a_{ij}$ in row $i$, column $j$ is
\[ a_{ij}=\left\{\begin{array}{ll}
          r^{-1/2}&\quad(j=1)  \\ \{j(j-1)\}^{-1/2}   &\quad(i < j)  \\
          0       &\quad(i>j>1)\\ -(j-1)^{1/2}j^{-1/2}&\quad(i=j>1).
          \end{array}\right. \]
     It is also useful to write $\balpha_j$ for the (column) vector
     which consists of the $j$th column of the matrix $\matr A$.  Show 
     that if the variates $X_i$ are independently $\N(\theta_i, 1)$, 
     then the variates 
     $W_j=\balpha_j\transpose(\vect X-\bmu)=\sum_i a_{ij}(X_i-\mu_j)$
     are independently normally distributed with unit variance and 
     such that $\E W_j=0$ for $j>1$ and
\[ \vect W\transpose\vect W=\sum_j W_j^2=\sum_i (X_i-\mu_i)^2
   =(\vect X-\bmu)\transpose(\vect X-\bmu). \]
     By taking $a_{ij}\propto\theta_j-\mu_j$ for $i>j$, $a_{ij}=0$ for 
     $i < j$ and $a_{jj}$ such that $\sum_j a_{ij}=0$, extend this result 
     to the general case and show that 
     $\E\,W_1\propto\gamma=\sum_i(\theta_i-\mu_i)^2$.
     Deduce that the distribution of a non-central chi-squared variate
     depends only of $r$ and $\gamma$. 

\nextq Show that $R(\btheta,\bhthetajsplus) < R(\btheta,\bhthetajs)$ where
\[
\bhthetajsplus=\bmu+\max\left[\left(1-\frac{r-2}{S_1}\right),\,0\right]
                       (\vect X-\bmu) \]
(Lehmann 1983, Section 4.6, Theorem 6.2).

\nextq Writing 
\[ \est\btheta=(\matr A\transpose\matr A)^{-1}
                \matr A\transpose\vect x,\qquad
   \est\btheta_k=(\matr A\transpose\matr A+k\matr I)^{-1}
                  \matr A\transpose\vect x \]
     for the least-squares and ridge regression estimators for
     regression coefficients $\btheta$, show that
\[ \est\btheta-\est\btheta_k=k(\matr A\transpose\matr
A)^{-1}\est\btheta_k \]
     and that the bias of $\est\btheta_k$ is
\[ \vect b(k)=\{(\matr A\transpose\matr A+k\matr I)^{-1}
                 \matr A\transpose\matr A- \matr I\}\btheta \]
     while its variance-covariance matrix is 
\[ \Var\est\btheta_k=\phi(\matr A\transpose\matr A+k\matr I)^{-1}
   \matr A\transpose\matr A(\matr A\transpose\matr A+k\matr I)^{-1}. \]
     Deduce expressions for the sum $\mathcal G(k)$ of the squares of 
     the biases and for the sum $\mathcal F(k)$ of the variances of the
     regression coefficients, and hence show that the mean square error
     is
\[ MSE_k=\E(\est\theta_b-\theta)\transpose(\est\theta_k-\theta)
        =\mathcal F(k)+\mathcal G(k). \]
     Assuming that $\mathcal F(k)$ is continuous and monotonic
     decreasing with $\mathcal F^{\,\prime}(0)=0$ and that 
     $\mathcal G(k)$ is 
     continuous and monotonic increasing with $\mathcal G(k)=
     \mathcal G^{\,\prime}(k)=0$, deduce that there always exists a $k$ 
     such that $MSE_k < MSE_0$ (Theobald, 1974).

\nextq Show that the matrix $\matr H$ in Section 8.6
     satisfies $\matr B\transpose\matr H^{-1}\matr B=\matr 0$ and that
     if $\matr B$ is square and non-singular then $\matr H^{-1}$ vanishes.

\nextq Consider the following particular case of the two way layout.
     Suppose that eight plots are harvested on four of which one variety
     has been sown, while a different variety has been sown on the other
     four.  Of the four plots with each variety, two different
     fertilizers have been used on two each.  The yield will be 
     normally distributed with a mean $\theta$ dependent on the 
     fertiliser and the variety and with variance $\phi$.  It is 
     supposed \textit{a priori} that the mean 
     for plots yields sown with the two different varieties are 
     independently normally distributed with mean $\alpha$ and variance 
     $\psi_{\alpha}$, while the effect of the two different fertilizers 
     will add an amount which is independently normally distributed with
     mean $\beta$ and variance $\psi_{\beta}$.  This fits into the 
     situation described in Section 8.6 with $\Phi$ 
     being $\phi$ times an $8\times 8$ identity matrix and
\[ \matr A=
   \left(\begin{array}{cccc}1&0&1&0\\1&0&1&0\\1&0&0&1\\1&0&0&1\\
   0&1&1&0\\0&1&1&0\\0&1&0&1\\0&1&0&1\end{array}\right)
   ;\qquad\matr
B=\left(\begin{array}{cc}1&0\\1&0\\0&1\\0&1\end{array}\right)
   ;\qquad\BPsi=
   \left(\begin{array}{cccc}
         \psi_{\alpha}&0&0&0\\
         0&\psi_{\alpha}&0&0\\
         0&0&\psi_{\beta}&0\\
         0&0&0&\psi_{\beta}
         \end{array}\right).
\]
     Find the matrix $\matr K^{-1}$ needed to find the posterior of
     $\theta$.

\nextq Generalize the theory developed in Section 8.6
     to deal with the case where $\vect x\sim\N(\matr A\btheta,\BPhi)$ 
     and $\btheta\sim\N(\matr B\bmu,\BPsi)$ and knowledge of $\bmu$ 
     is vague to deal with the case where 
     $\bmu\sim\N(\matr C\bnu,\matr K)$\ (Lindley and Smith, 1972).

\nextq Find the elements of the variance-covariance matrix $\BSigma$ 
     for the one way model in the case where $n_i=n$ for all $i$.

\section{Exercises on Chapter \arabic{section}}

\setcounter{qno}{0}

\nextq Find the value of $\int_0^1 \text{e}^x\dx$ by crude Monte Carlo
     integration using a sample size of $n=10$ values from a uniform
     distribution $\U(0, 1)$ taken from tables of random numbers [use,
     for example, groups of random digits from Lindley and Scott (1995,
     Table 27) or Neave (1978, Table 8.1)].  Repeat the experiment 10 
     times and compute the overall mean and the sample standard
     deviation of the values you obtain.  What is the theoretical 
     value of the population standard deviation and how does the 
     value you obtained compare with it?

\nextq Suppose that, in a Markov chain with just two states, the 
     probabilities of going from state $i$ to state $j$ in one time unit
     are given by the entries of the matrix
\[ \matr A=\left(\begin{array}{cc}
   \slopefrac{1}{4}&\slopefrac{3}{4}\\ \slopefrac{1}{2}&\slopefrac{1}{2}
   \end{array}\right) \]
     in which $i$ represents the row and $j$ the column.  Show that the 
     probability of getting from state $i$ to state $j$ in $t$ time
     units is given by the $t$th power of the matrix $\matr A$ and that
\[ \matr A^t=\left(\begin{array}{cc}
   \slopefrac{2}{5}&\slopefrac{3}{5}\\ \slopefrac{2}{5}&\slopefrac{3}{5}
   \end{array}\right)
   +\left(\mbox{\large{$-\frac{1}{4}$}}\right)^t
   \left(\begin{array}{cc}
  
\slopefrac{3}{5}&-\slopefrac{3}{5}\\-\slopefrac{2}{5}&\slopefrac{2}{5}
   \end{array}\right). \]
     Deduce that, irrespective of the state the chain started in, after
     a long time it will be in the first state with probability
     $\slopefrac{2}{5}$ and in the second state with probability 
     $\slopefrac{3}{5}$.

\nextq Smith (1969, Section 21.10) quotes an example on genetic linkage 
     in which we have observations $\vect x=(x_1, x_2, x_3, x_4)$ with
     cell probabilities
\[ \left(\quarter+\quarter\eta,\,\quarter\eta,\,
         \quarter(1-\eta),\,\quarter(1-\eta)+\quarter\right). \]
     The values quoted are $x_1=461$, $x_2=130$, $x_3=161$ and
     $x_4=515$.  Divide $x_1$ into $y_0$ and $y_1$ and $x_4$ 
     into $y_4$ and $y_5$ 
     to produce augmented data $\vect y=(y_0, y_1, y_2, y_3, y_4, y_5)$ 
     and use the \EM\ al\-gorithm to estimate $\eta$.
  
\nextq Dempster \textit{et al.}\ (1977) define a generalized \EM\
     al\-gorithm (abbreviated as a \GEM\ al\-gorithm) as one in which
     $Q(\theta^{(t+1)},\theta^{(t)})\geqslant
     Q(\theta^{(t)},\theta^{(t)})$.  
     Give reasons for believing that \GEM\ al\-gorithms converge to the 
     posterior mode.

\nextq In question 16 in Chapter 2
     we supposed that the results of a certain test were known, on the 
     basis of general theory, to be normally distributed about the same 
     mean $\mu$ with the same variance $\phi$, neither of which is
     known.  In that question we went on to suppose that your prior 
     beliefs about 
     $(\mu, \phi)$ could be represented by a normal/chi-squared 
     distribution with
\[   \nu_0  =  4, \qquad S_0  = 350, \qquad n_0 = 1, \qquad \theta_0  = 
      85. \]
     Find a semi-conjugate prior which has marginal distributions that
     are close to the marginal distributions of the normal/chi-squared
     prior but is such that the mean and variance are independent
     \textit{a priori}.  Now suppose as previously that 100 observations
     are obtained from the population with mean 89 and sample variance 
     $s^2 = 30$.  Find the posterior distribution of $(\mu, \phi)$.  
     Compare the posterior mean obtained by the \EM\ al\-gorithm with
     that obtained from the fully conjugate prior.

\nextq A textile company weaves a fabric on a large number of looms.
     Four looms selected at random from those available, and four 
     observations of the tensile strength of fabric woven on each of 
     these looms are available (there is no significance to the order 
     of the observations from each of the looms), and the resulting 
     data are given below:
\[
\begin{array}{ccccc}
  \text{Loom}&\multicolumn{4}{c}{\qquad\text{Observations}} \\
        1     &\qquad 98 \quad&\quad 97 \quad&\quad 99 \quad&\quad 96 
\\
        2     &\qquad 91 \quad&\quad 90 \quad&\quad 93 \quad&\quad 92 
\\
        3     &\qquad 96 \quad&\quad 95 \quad&\quad 97 \quad&\quad 95 
\\
        4     &\qquad 95 \quad&\quad 96 \quad&\quad 99 \quad&\quad 98
\end{array}
\]
     Estimate the means for each of the looms, the overall mean, the 
     variance of observations from the same loom, and the variance of 
     means from different looms in the population.

\nextq Write computer programs in C++ equivalent to the programs in
    \R\ in this chapter.

\nextq Use the data augmentation al\-gorithm to estimate the posterior
     density of the parameter $\eta$ in the linkage model in question
     3 above.

\nextq Suppose that $y\,|\,\pi\sim\B(n,\pi)$ and
     $\pi\,|\,y\sim\Be(y+\alpha,\,n-y+\beta)$ where $n$ is a Poisson
     variable of mean $\lambda$ as opposed to being fixed as in Section 
     9.4.  Use the Gibbs sampler (chained data augmentation)
     to find the unconditional distribution of $n$ in the case where 
     $\lambda=16$. $\alpha=2$ and $\beta=4$ (cf.\ Casella and George, 
     1992).

\nextq Find the mean and variance of the posterior distribution of
     $\theta$ for the data in question 5 above using the
     prior you derived in answer to that question by means of the Gibbs
     sampler (chained data augmentation).

\nextq The data below represent the weights of $r=30$ young rats
     measured weekly for $n=5$ weeks as quoted by Gelfand 
     \textit{et al.}\ (1990), Tanner (1996, Table 1.3 and Section 
     6.2.1), Carlin and Louis (2000, Example 5.6):
{\catcode`?=\active
  \def?{\kern\digitwidth}
  \small
\[
\begin{array}{cccccccccccc}
 \text{Rat$\backslash$Week}&1&2&3&4&5&\text{Rat$\backslash$Week}&1&2&3&4&5
\\
  ?1\qquad& 151&199&246&283&320& 16\qquad& 160&207&248&288&324 \\
  ?2\qquad& 145&199&249&293&354& 17\qquad& 142&187&234&280&316 \\
  ?3\qquad& 147&214&263&312&328& 18\qquad& 156&203&243&283&317 \\
  ?4\qquad& 155&200&237&272&297& 19\qquad& 157&212&259&307&336 \\
  ?5\qquad& 135&188&230&280&323& 20\qquad& 152&203&246&286&321 \\
  ?6\qquad& 159&210&252&298&331& 21\qquad& 154&205&253&298&334 \\
  ?7\qquad& 141&189&231&275&305& 22\qquad& 139&190&225&267&302 \\
  ?8\qquad& 159&201&248&297&338& 23\qquad& 146&191&229&272&302 \\
  ?9\qquad& 177&236&285&340&376& 24\qquad& 157&211&250&285&323 \\
  10\qquad& 134&182&220&260&296& 25\qquad& 132&185&237&286&331 \\
  11\qquad& 160&208&261&313&352& 26\qquad& 160&207&257&303&345 \\
  12\qquad& 143&188&220&273&314& 27\qquad& 169&216&261&295&333 \\
  13\qquad& 154&200&244&289&325& 28\qquad& 157&205&248&289&316 \\
  14\qquad& 171&221&270&326&358& 29\qquad& 137&180&219&258&291 \\
  15\qquad& 163&216&242&281&312& 30\qquad& 153&200&244&286&324
\end{array}
\]
}
     The weight of the $i$th rat in week $j$ is denoted $x_{ij}$ and
     we suppose that weight growth is linear, that is,
\[    x_{ij}\sim\N(\alpha_i+\beta_i j,\,\phi), \]
     but that the slope and intercept vary from rat to rat.  We further
     suppose that $\alpha_i$ and $\beta_i$ have a bivariate normal
     distribution, so that
\[    \btheta_i=\left(\begin{array}{c}\alpha_i\\
\beta_i\end{array}\right)
      \sim\N(\btheta_0,\,\BSigma)\quad\text{where}\quad
      \btheta_0=\left(\begin{array}{c}\alpha_0\\
\beta_0\end{array}\right),
\]
     and thus we have a random effects model.  At the third stage, we
     suppose that
\[    p(\matr V\,|\,\nu, \BOmega)\propto
      \frac{|\matr V|^{(\nu-k-1)/2}}{|\BOmega|^{\nu/2}}
      \exp\left[-\half\text{Trace}(\BOmega^{-1}\matr V)\right]. \]
     Methods of sampling from this distribution are described in Odell
     and Feiveson (1966), Kennedy and Gentle (1990, Section 6.5.10) and 
     Gelfand \textit{et al.}\ (1990).  [This example was omitted from 
     the main text because we have avoided use of the Wishart
     distribution elsewhere in the book.  A slightly simpler model 
     in which $\BSigma$ is assumed to be diagonal is to be found as 
     the example `Rats' distributed with WinBUGS.]
     
     Explain in detail how you would use the Gibbs sampler to estimate
     the posterior distributions of $\alpha_0$ and $\beta_0$, and if
     possible carry out this procedure.

\nextq Use the Met\-ropo\-lis-Hast\-ings al\-gorithm to estimate the 
     posterior density of the parameter $\eta$ in the linkage model in
     Sections 9.2 and 9.3 using candidate values 
     generated from a uniform distribution on $(0, 1)$ [cf.\ Tanner  
     (1996, Section 6.5.2)].

\nextq Write a WinBUGS program to analyze the data on wheat yield
     considered towards the end of Section 2.13 and in Section 9.3.

\nextq In bioassays the response may vary with a covariate termed the
     \textit{dose}.  A typical example involving a binary response is 
     given in the table below, where $R$ is the number of beetles 
     killed after five hours exposure to gaseous carbon disulphide at 
     various concentrations (data from Bliss, 1935, quoted by Dobson, 
     2002, Example 7.3.1).
\begin{center}
  \begin{tabular}{lll}
    \hline
    \multicolumn{1}{c}{Dose $x_i$} & Number of & Number \\
    ($\text{log}_{10} \text{CS}_2 \text{mg l}^{-2})$
    & insects, $n_i$ & killed, $r_i$ \\
    \hline
    \qquad 1.6907 & \quad 59 & \quad \phantom{1}6 \\
    \qquad 1.7242 & \quad 60 & \quad 13 \\
    \qquad 1.7552 & \quad 62 & \quad 18 \\
    \qquad 1.7842 & \quad 56 & \quad 28 \\
    \qquad 1.8113 & \quad 63 & \quad 52 \\
    \qquad 1.8369 & \quad 59 & \quad 53 \\
    \qquad 1.8610 & \quad 62 & \quad 61 \\
    \qquad 1.8839 & \quad 60 & \quad 60 \\
    \hline
  \end{tabular}
\end{center}

\noindent
     Fit a logistic regression model and plot the proportion killed against
     dose and the fitted line.

\section{Exercises on Chapter \arabic{section}}

\startqs

\nextq Suppose that $x \sim \C(0,1)$ has a Cauchy distribution.  It is
       easily shown that $\eta = \Pr(x > 2) = \tan^{-1}(\half)/\pi =
       0.147\,583\,6$, but we will consider Monte Carlo methods of 
       evaluating this probability.
       \begin{description}
         \item[\quad(a)] Show that if $k$ is the number of values taken
           from a  random sample of size $n$ with a Cauchy distribution,
           then $k/n$ is an estimate with variance $0.125\,802\,7/n$.
         \item[\quad(b)] Let $p(x)=2/x^2$ so that $\int_x^{\infty}p(\xi)
           \dxi = 2/x$.  Show that if $x \sim \U(0,1)$ is uniformly 
           distributed over the unit interval then $y=2/x$ has the 
           density $p(x)$ and that all values of $y$ satisfy $y\geqslant
           2$ and hence that
\[ \sum_{i=1}^n \frac{1}{2\pi}\frac{y_i^2}{1+y_i^2} \]
           gives an estimate of $\eta$ by importance sampling. 
         \item[\quad(c)] Deduce that if $x_1$, $x_2$, \dots, $x_n$ are
           independent $\U(0,1)$ variates then
\[ \widehat\eta =
   \frac{1}{n}\sum_{i=1}^n\frac{1}{2\pi}\frac{4}{4+x_i^2} \]
           gives an estimate of $\eta$.
         \item[\quad(d)] Check that $\widehat\eta$ is an unbiased 
           estimate of $\eta$ and show that
\[ \E\widehat\eta^2 = \frac{\tan^{-1}(\half)+\twofifths}{4\pi^2} \]
           and deduce that 
\[ \Var\widehat\eta = 0.000\,095\,5 \]
           so that this estimator has a notably smaller variance than 
           the estimate considered in (a).
       \end{description}

\nextq Apply sampling importance resampling starting from random 
       variables uniformly distributed over $(0,1)$ to estimate 
       the mean and variance of a beta distribution $\Be(2,3)$.

\nextq Use the sample found in the previous section to find a 90\% HDR 
       for $\Be(2,3)$ and compare the resultant limits with the values
       found using the methodology of Section 3.1.  Why do the values 
       differ?

\nextq Apply the methodology used in the numerical example in Section
       \ref{sec:varbayes} to the dataset used in both Exercise 16 on 
       Chapter 2 and Exercise 5 on Chapter 9.

\nextq Find the Kullback-Leibler divergence $\I(q:p)$ when $p$ is a
       binomial distribution $\B(n,\pi)$ and $q$ is a binomial 
       distribution $\B(n,\rho)$.  When does $\I(q:p)=\I(p:q)$?

\nextq Find the Kullback-Leibler divergence $\I(q:p)$ when $p$ is a
       normal distribution $\N(\mu,\phi)$ and $q$ is a normal 
       distribution $\N(\nu,\psi)$. 

\nextq Let $p$ be the density $2(2\pi)^{-1/2}\exp(-\half x^2)$ $(x>0)$
       of the modulus $|z|$ of a standard normal distribution and let
       $q$ be the density $\beta^{-1}\exp(-x/\beta)$ $(x>0)$ of an 
       $\Ex(\beta)$ distribution.  Find the value of $\beta$ such 
       that $q$ is as close an approximation to $p$ as possible in
       the sense that $\I(q:p)$ is a minimum.

\nextq The paper by Corduneaunu and Bishop (2001) referred to in Section
       \ref{sec:varbayesgeneral} can be found on the web at
{\small
\[
\texttt{http://research.microsoft.com/pubs/67239/bishop-aistats01.pdf.}
\]
}
       \!\!H\"ardle's data set is available in \R\ by going 
       \texttt{data(faithful)}.  Fill in the details of the analysis of 
       a mixture of multivariate normals given in that section.

\nextq Carry out the calculations in Section 10.4 for the genetic 
       linkage data quoted by Smith which was given in Exercise 3 
       on Chapter 9.

\nextq A group of $n$ students sit two exams. Exam one is 
       on history and exam two is on chemistry.  Let $x_i$ and $y_i$
       denote the $i$th student's score in the history and chemistry
       exams, respectively. The following linear regression model is
       proposed for the relationship between the two exam scores:
\[ y_i = \alpha + \beta x_i + \varepsilon_i\quad (i = 1, 2, \dots, n) \]
       where $\varepsilon_i \sim \N(0,1/\tau)$.

       Assume that $\alpha$, $\beta$ and $\tau$ are unknown parameters
       to be estimated and $\vect x = (x_1, x_2, \dots, x_n)$ and
       $\vect y = (y_1, y_2, \dots, y_n)$.

       Describe a reversible jump \MCMC\ algorithm including discussion 
       of the acceptance probability, to move between the four competing
       models:
       \begin{enumerate}
         \item $y_i = \alpha + \varepsilon_i$;
         \item $y_i = \alpha + \beta x_i + \varepsilon_i$;
         \item $y_i = \alpha + \lambda t_i + \varepsilon_i$;
         \item $y_i = \alpha + \beta x_i + \lambda t_i + \varepsilon_i$.
       \end{enumerate}
       Note that if $z$ is a random variable with probability density 
       function $f$ given by
\[ f(z) \propto \exp\left(-\half A\left(z^2-2Bz\right)\right), \]
       then $z \sim N(B,1/A)$ [due to P.~Neal].

\end{document}

%